Степень окисления pbo: p2 o3, p2o5, pbo, pbo2,n2o5,n2o3 — Знания.org

Содержание

степень окисления, химические свойства, формула, применение

Свинец (Pb) – мягкий серебристо-белый или сероватый металл 14-й (IVa) группы периодической таблицы с атомным номером 82. Это очень податливое, пластичное и плотное вещество, которое плохо проводит электричество. Электронная формула свинца – [Хе] 4f14 5d10 6s2 6p2. Известный в древности и считавшийся алхимиками старейшим из металлов, он очень долговечен и устойчив к коррозии, о чем свидетельствует продолжающаяся эксплуатация водопроводных труб, установленных древними римлянами. Символ Pb в химической формуле свинца является аббревиатурой латинского слова plumbum.

Распространенность в природе

Свинец часто упоминается в ранних библейских текстах. Вавилоняне использовали металл для изготовления пластин для письма. Римляне делали из него водопроводные трубы, монеты и даже кухонные принадлежности. Результатом последнего стало отравление населения свинцом в эпоху императора Августа Цезаря. Соединение, известное как белый свинец, использовалось в качестве декоративного пигмента еще в 200 г. до н. э.

В весовом отношении содержание свинца в земной коре соответствует олову. В космосе на 106 атомов кремния приходится 0,47 атома свинца. Это сопоставимо с содержанием цезия, празеодима, гафния и вольфрама, каждый из которых рассматривается как достаточно дефицитный элемент.

Добыча

Хотя свинца не так много, процессы естественной концентрации привели к значительным залежам коммерческого значения, особенно в Соединенных Штатах, Канаде, Австралии, Испании, Германии, Африке и Южной Америке. Редко встречающийся в чистом виде свинец присутствует в нескольких минералах, но все они имеют второстепенное значение, за исключением сульфида PbS (галенита), который является основным источником промышленного производства данного химического элемента во всем мире. Металл также содержится в англезите (PbSO4) и церуссите (PbCO3). К началу XXI в. ведущими мировыми производителями концентрата свинца являлись такие страны, как Китай, Австралия, США, Перу, Мексика и Индия.

Свинец может быть извлечен путем обжига руды с последующей плавкой в доменной печи или методом прямой плавки. Примеси удаляются в ходе дополнительной очистки. Почти половина всего очищенного свинца извлекается из переработанного лома.

Химические свойства

Элементарный свинец может быть окислен до иона Pb2+ ионами водорода, но нерастворимость его большинства солей делает этот химический элемент устойчивым к воздействию многих кислот. Окисление в щелочной среде происходит легче и благоприятствует образованию растворимых соединений при степени окисления свинца +2. Оксид PbO2 с ионом Pb4+ является одним из наиболее сильных окислителей в кислом растворе, но он сравнительно слабый в щелочном растворе. Окисление свинца облегчается путем образования комплексов. Электроосаждение лучше всего проводить из водных растворов, содержащих гексафторсиликат свинца и гексафторсиликатную кислоту.

На воздухе металл быстро окисляется, образуя тусклое серое покрытие, ранее считавшееся субоксидом Pb2O. Теперь общепризнано, что это смесь Pb и оксида PbO, который защищает металл от дальнейшей коррозии. Хотя свинец растворяется в разбавленной азотной кислоте, он лишь поверхностно подвергается воздействию соляной или серной кислот, потому что образующиеся нерастворимые хлориды (PbCl2) или сульфаты (PbSO4) предотвращают продолжение реакции. Химические свойства свинца, обуславливающие его общую стойкость, позволяют использовать металл для изготовления кровельных материалов, оболочки электрических кабелей, размещенных в грунте или под водой, и в качестве прокладки для водопроводных труб и конструкций, служащих для транспортировки и переработки коррозионных веществ.

Применение свинца

Известна только одна кристаллическая модификация данного химического элемента с плотно упакованной металлической решеткой. В свободном состоянии проявляется нулевая степень окисления свинца (как и любого другого вещества). Широкое применение элементарной формы элемента обусловлено ее пластичностью, легкостью сварки, низкой температурой плавления, высокой плотностью и способностью поглощать гамма- и рентгеновское излучение. Расплавленный свинец является отличным растворителем и позволяет концентрировать свободное серебро и золото. Конструкционное применение свинца ограничено его низкой прочностью на растяжение, усталостью и текучестью даже при малой нагрузке.

Элемент находит применение в производстве аккумуляторных батарей, в боеприпасах (выстрелах и пулях), в составе припоя, типографском, подшипниковых, легких сплавах и сплавах с оловом. В тяжелом и промышленном оборудовании для снижения шума и вибрации могут использоваться детали из соединений свинца. Поскольку металл эффективно поглощает коротковолновое электромагнитное излучение, он применяется для защитного экранирования ядерных реакторов, ускорителей частиц, рентгеновского оборудования и контейнеров для транспортировки и хранения радиоактивных материалов. В составе оксида (PbO2) и сплава с сурьмой или кальцием элемент используется в обычных аккумуляторных батареях.

Действие на организм

Химический элемент свинец и его соединения токсичны и накапливаются в организме в течение длительного периода времени (данное явление известно как кумулятивное отравление) до достижения смертельной дозы. Токсичность возрастает по мере увеличения растворимости соединений. У детей накопление свинца может привести к когнитивным расстройствам. У взрослых оно вызывает прогрессирующую болезнь почек. К симптомам отравления относятся боль в животе и диарея, за которыми следуют запоры, тошнота, рвота, головокружение, головная боль и общая слабость. Устранение контакта с источником свинца обычно является достаточным для лечения. Устранение химического элемента из инсектицидов и пигментных красок, а также использование респираторов и других защитных устройств в местах воздействия значительно уменьшили число случаев отравления свинцом. Признание того, что тетраэтилсвинец Pb (C2H5)4 в виде антидетонационной присадки к бензину загрязняет воздух и воду, привело к прекращению его применения в 1980-х годах.

Биологическая роль

Свинец не играет никакой биологической роли в организме. Токсичность этого химического элемента вызвана его способностью имитировать такие металлы, как кальций, железо и цинк. Взаимодействие свинца с теми же молекулами белка, что и эти металлы, приводит к прекращению их нормального функционирования.

Ядерные свойства

Химический элемент свинец образуется как в результате нейтронно-абсорбционных процессов, так и при распаде радионуклидов более тяжелых элементов. Существуют 4 стабильных изотопа. Относительная распространенность 204Pb составляет 1,48 %, 206Pb – 23,6 %, 207Pb – 22,6 % и 208Pb – 52,3 %. Стабильные нуклиды являются конечными продуктами естественного радиоактивного распада урана (до 206Pb), тория (до 208Pb) и актиния (до 207Pb). Известно более 30 радиоактивных изотопа свинца. Из них в процессах естественного распада участвуют 212Pb (ряд тория), 214Pb и 210Pb (серия урана) и 211Pb (ряд актиния). Атомный вес естественного свинца варьируется от источника к источнику в зависимости от его происхождения.

Монооксиды

В соединениях степени окисления свинца в основном равны +2 и +4. В число наиболее важных из них входят оксиды. Это PbO, в котором химический элемент находится в состоянии +2, диоксид PbO2, в котором проявляется высшая степень окисления свинца (+4), и тетраоксид, Pb3O4.

Монооксид существует в двух модификациях – литарга и глета. Литарг (альфа-оксид свинца) представляет собой красное или красновато-желтое твердое вещество с тетрагональной кристаллической структурой, стабильная форма которого существует при температурах ниже 488 °С. Глет (бета-монооксид свинца) является желтым твердым веществом и имеет орторомбическую кристаллическую структуру. Его стабильная форма существует при температурах, превышающих 488 °C.

Обе формы нерастворимы в воде, но растворяются в кислотах с образованием солей, содержащих ион Pb2+ или в щелочах с образованием плюмбитов, которые имеют PbO22--ион. Литарг, который образуется при реакции свинца с кислородом воздуха, является самым важным коммерческим соединением этого химического элемента. Вещество используется в больших количествах непосредственно и как исходный материал для получения других соединений свинца.

Значительное количество PbO расходуется при изготовлении пластин свинцово-кислотных аккумуляторных батарей. Высококачественная стеклянная посуда (хрусталь) содержит до 30 % литарга. Это увеличивает показатель преломления стекла и делает его блестящим, прочным и звонким. Литарг также служит осушителем в лаках и применяется в производстве свинцовокислого натрия, который используется для удаления из бензина неприятно пахнущих тиолов (органических соединений, содержащих серу).

Диоксид

В природе PbO2 существует в виде коричнево-черного минерала платтнерита, который коммерчески производится из тетраоксида триалада окислением хлором. Он разлагается при нагревании и дает кислород и оксиды с более низкой степенью окисления свинца. PbO2 используется в качестве окислителя при производстве красителей, химикатов, пиротехники и спиртов и как отвердитель для полисульфидных каучуков.

Тетраоксид трисвинца Pb3O4 (известный как свинцовый сурик или миниум) получают путем дальнейшего окисления PbO. Это пигмент от оранжево-красного до кирпично-красного цвета, который входит в состав коррозионностойких красок, применяемых для защиты подверженных воздействию окружающей среды железа и стали. Он также реагирует с оксидом железа с образованием феррита, используемого при изготовлении постоянных магнитов.

Ацетат

Также экономически значимым соединением свинца степени окисления +2 является ацетат Pb(C2H3O2)2. Это водорастворимая соль, получаемая путем растворения глета в концентрированной уксусной кислоте. Общая форма, тригидрат, Pb(C2H3O2)2 · 3H2O, называемый сахаром свинца, используется в качестве закрепителя при окрашивании тканей и как сиккатив в некоторых красках. Кроме того, он применяется в производстве других соединений свинца и в установках для цианирования золота, где он в виде PbS служит для осаждения из раствора растворимых сульфидов.

Другие соли

Основные карбонат, сульфат и силикат свинца когда-то широко применялись в качестве пигментов для белых красок наружного применения. Однако с середины ХХ в. использование т. н. белых свинцовых пигментов значительно уменьшилось из-за беспокойства по поводу их токсичности и сопутствующей опасности для здоровья человека. По той же причине практически прекратилось применение арсената свинца в инсектицидах.

Помимо основных состояний окисления (+4 и +2) свинец может иметь отрицательные степени -4, -2, -1 в фазах Цинтля (например, BaPb, Na8Ba8Pb6), а +1 и +3 – в свинецорганических соединениях, таких как гексаметилдиплюмбан Pb2(CH3)6.

1.4. Оксиды — ЗФТШ, МФТИ

Многие элементы проявляют переменную валентность и дают оксиды различного состава, что учитывается по международной номенклатуре  с указанием степени окисления элемента римскими цифрами, например, `»SO»_3` — оксид серы (VI),  `»SO»_2` — оксид серы (IV).

Можно представить следующую схему:

Название оксида  `=` «Оксид» `+` название элемента в род. падеже `+` (валентность римскими цифрами).

По своим химическим свойствам все оксиды подразделяются на солеобразующие и несолеобразующие. Солеобразующие оксиды принято делить на три основные группы: основные, амфотерные и кислотные.

Получение основных оксидов

а) разложение нерастворимых гидроксидов:  

нерастворимые основания при нагревании разлагаются на соответствующий оксид и воду:

$$ \mathrm{Cu}{\left(\mathrm{OH}\right)}_{2}\stackrel{\mathrm{t}°\mathrm{C}}{\to }\mathrm{CuO}+{\mathrm{H}}_{2}\mathrm{O}$$.

Этим методом нельзя получить гидроксиды щелочных металлов.

б) разложение солей кислородсодержащих  кислот:

  • разложение карбонатов:

карбонаты щелочноземельных металлов и магния разлагаются на соответствующие основный и кислотный оксиды:

$$ {\mathrm{BaCO}}_{3}\stackrel{\mathrm{t}°\mathrm{C}}{\to }\mathrm{BaO}+{\mathrm{CO}}_{2}\uparrow $$.

Ниже приведены уравнения реакций разложения карбоната аммония и карбоната серебра:

$$ {\left({\mathrm{NH}}_{4}\right)}_{2}{\mathrm{CO}}_{3}\stackrel{t°\mathrm{C}}{\to }2{\mathrm{NH}}_{3}\uparrow +{\mathrm{CO}}_{2}+{\mathrm{H}}_{2}\mathrm{O}$$;

$$ 2{\mathrm{Ag}}_{2}{\mathrm{CO}}_{3}\stackrel{t°\mathrm{C}}{\to }4\mathrm{Ag}\downarrow +2{\mathrm{CO}}_{2}\uparrow +{\mathrm{O}}_{2}\uparrow $$.

Карбонаты щелочных металлов плавятся без разложения, исключение составляет карбонат лития :

$$ {\mathrm{Li}}_{2}{\mathrm{CO}}_{3}\stackrel{\mathrm{t}°\mathrm{C}}{\to }{\mathrm{Li}}_{2}\mathrm{O}+{\mathrm{CO}}_{2}\uparrow $$.

  • разложение гидрокарбонатов металлов:

гидрокарбонаты разлагаются на карбонат, воду и углекислый газ.

$$ 2{\mathrm{NaHCO}}_{3}\stackrel{\mathrm{t}°\mathrm{C}}{\to }{\mathrm{Na}}_{2}{\mathrm{CO}}_{3} + {\mathrm{CO}}_{2}\uparrow  + {\mathrm{H}}_{2}\mathrm{O}$$.

  • разложение нитратов

разложение нитратов металлов определяется положением металла в электрохимическом ряду напряжений металлов:

$$ 2{\mathrm{KNO}}_{3} \stackrel{\mathrm{t}°\mathrm{C}}{\to }2{\mathrm{KNO}}_{2} + {\mathrm{O}}_{2}\uparrow $$

(если металл в ряду напряжения стоит до магния),

$$ 2\mathrm{Pb}({\mathrm{NO}}_{3}{)}_{2} \stackrel{\mathrm{t}°\mathrm{C}}{\to }  2\mathrm{PbO} + 4{\mathrm{NO}}_{2}\uparrow + {\mathrm{O}}_{2}\uparrow $$

(если металл в ряду напряжения стоит от магния до меди включительно),

$$ 2{\mathrm{AgNO}}_{3}\stackrel{\mathrm{t}°\mathrm{C}}{\to } 2\mathrm{Ag} + 2{\mathrm{NO}}_{2}\uparrow  + {\mathrm{O}}_{2}\uparrow $$

(если металл в ряду напряжения стоит после меди).

ВНИМАНИЕ!!!

$$ {\mathrm{NH}}_{4}{\mathrm{NO}}_{3} \stackrel{\mathrm{t}°\mathrm{C}}{\to } {\mathrm{N}}_{2}\mathrm{O}\uparrow  + 2{\mathrm{H}}_{2}\mathrm{O}$$;

$$ 4{\mathrm{LiNO}}_{3}\stackrel{t°\mathrm{C}}{\to }2\mathrm{LiO}+4{\mathrm{NO}}_{2}\uparrow +{\mathrm{O}}_{2}\uparrow $$;

$$ 4\mathrm{Fe}{\left({\mathrm{NO}}_{3}\right)}_{2}\stackrel{t°\mathrm{C}}{\to }2{\mathrm{Fe}}_{2}{\mathrm{O}}_{3}+8{\mathrm{NO}}_{2}\uparrow +{\mathrm{O}}_{2}\uparrow $$;

$$ \mathrm{Mn}{\left({\mathrm{NO}}_{3}\right)}_{2}\stackrel{\mathrm{t}°\mathrm{C}}{\to }{\mathrm{MnO}}_{2}\downarrow +2{\mathrm{NO}}_{2}\uparrow $$.

  • разложение основных солей:

$$ (\mathrm{ZnOH}{)}_{2}{\mathrm{CO}}_{3} \stackrel{\mathrm{t}°\mathrm{C}}{\to } 2\mathrm{ZnO} + {\mathrm{CO}}_{2}\uparrow  + {\mathrm{H}}_{2}\mathrm{O}$$.

в) разложение кислородсодержащих кислот:

                   `»H»_2″SO»_3  → «SO»_2 + «H»_2″O»`.

г) обработка солей растворами кислот:

 `»K»_2″Cr»_2″O»_7 + «H»_2″SO»_4  →2″CrO»_3+»K»_2″SO»_4+»H»_2″O»`,

`»Na»_2″SiO»_3+2″HCl» -> 2″NaCl»+»SiO»_2+»H»_2″O»`.

Химические свойства основных оксидов

Основные оксиды при нагревании могут вступать в реакции с кислотными и амфотерными оксидами, амфотерными гидроксидами, с кислотами.

Большинство кислотных оксидов непосредственно взаимодействуют с водой с образованием кислот (исключение `»SiO»_2`):

`»P»_2″O»_5+3″H»_2″O» -> 2″H»_3″PO»_4`;

Оксиды, которым соответствуют неустойчивые кислоты, реагируют с водой обратимо и в очень малой степени.

$$ {\mathrm{CO}}_{2} + {\mathrm{H}}_{2}\mathrm{O}\rightleftarrows {\mathrm{H}}_{2}{\mathrm{CO}}_{3}$$;

$$ {\mathrm{SO}}_{2} + {\mathrm{H}}_{2}\mathrm{O}\rightleftarrows {\mathrm{H}}_{2}{\mathrm{SO}}_{3}$$.

Азот в оксидах способен проявлять  в соединениях степени окисления `+1`, `+2`, `+3`, `+4`, `+5`. Оксиды `»N»_2″O»` и `»NO»` является несолеобразующими оксидами, оксиды `»N»_2″O»_3` и `»N»_2″O»_5` являются кислотными оксидами, которым соответствуют азотистая и азотная кислота соответственно:

$$ {\mathrm{N}}_{2}{\mathrm{O}}_{3} + {\mathrm{H}}_{2}\mathrm{O} \to  2{\mathrm{HNO}}_{2}$$;

$$ {\mathrm{N}}_{2}{\mathrm{O}}_{5} + {\mathrm{H}}_{2}\mathrm{O} \to  2{\mathrm{HNO}}_{3}$$.

Оксид `»NO»_2` является кислотным оксидом, которому соответствуют сразу две кислоты:

$$ 2{\mathrm{NO}}_{2} + {\mathrm{H}}_{2}\mathrm{O} \to  {\mathrm{HNO}}_{2} + {\mathrm{HNO}}_{3} $$.

Наряду с современной номенклатурой для кислотных оксидов до сих пор широко используется старинная система названий как ангидридов кислот – продуктов отщепления воды от соответствующих кислот. Например, `»SO»_3` – ангидрид серной кислоты, `»SO»_2` – ангидрид сернистой кислоты. Наиболее типичными для кислотных оксидов являются их реакции с основными и амфотерными оксидами, амфотерными гидроксидами, щелочами:

$$ {\mathrm{SO}}_{3} + \mathrm{CuO} \to  {\mathrm{CuSO}}_{4}$$;

$$ {\mathrm{SO}}_{3} + {\mathrm{Na}}_{2}\mathrm{O} \to  {\mathrm{Na}}_{2}{\mathrm{SO}}_{4}$$.

$$ {\mathrm{P}}_{2}{\mathrm{O}}_{5}+ {\mathrm{Al}}_{2}{\mathrm{O}}_{3} \stackrel{\mathrm{t}°\mathrm{C}}{\to } 2{\mathrm{AlPO}}_{4}$$;

$$ 3{\mathrm{SO}}_{3} + 2\mathrm{Al}(\mathrm{OH}{)}_{3} \to  {\mathrm{Al}}_{2}({\mathrm{SO}}_{4}{)}_{3} +3{\mathrm{H}}_{2}\mathrm{O}$$.

$$ \mathrm{Ca}(\mathrm{OH}{)}_{2} + {\mathrm{CO}}_{2} \to  {\mathrm{CaCO}}_{3} \downarrow  + {\mathrm{H}}_{2}\mathrm{O}$$;

$$ {\mathrm{SO}}_{3} + 2\mathrm{NaOH} \to  {\mathrm{Na}}_{2}{\mathrm{SO}}_{4} + {\mathrm{H}}_{2}\mathrm{O}$$;

$$ {\mathrm{SO}}_{3} + \mathrm{NaOH} \to  {\mathrm{NaHSO}}_{4}$$.

Кислотные оксиды слабых кислот взаимодействуют только с сильными основаниями (щелочами) и их оксидами:

$$ {\mathrm{Na}}_{2}\mathrm{O}+{\mathrm{CO}}_{2}\to {\mathrm{Na}}_{2}{\mathrm{CO}}_{3}$$;

`»CuO»+»CO»_2`$$ \overline{)\to }$$.  

Выше упоминалось, что кислотные оксиды могут вступать в многочисленные окислительно-восстановительные реакции, например:

$$ {\mathrm{CO}}_{2} + \mathrm{C} \to  2\mathrm{CO}\uparrow $$;

$$ {\mathrm{SO}}_{2} + 2{\mathrm{H}}_{2}\mathrm{S}  \to  3\mathrm{S}\downarrow + 2{\mathrm{H}}_{2}\mathrm{O}$$.

В состав амфотерного окcида входит элемент, который обладает амфотерными свойствами. Под амфотерностью понимают способность соединений проявлять в зависимости от условий кислотные и основные свойства. Например, оксид цинка `»ZnO»` может быть как основанием, так и кислотой (`»Zn»(«OH»)_2` и `»H»_2″ZnO»_2`). Амфотерность выражается в том, что в зависимости от условий амфотерные оксиды проявляют либо осно́вные, либо кислотные свойства:

$$ \mathrm{ZnO} + {\mathrm{CO}}_{2} \to  {\mathrm{ZnCO}}_{3}$$;
$$ \mathrm{ZnO} + {\mathrm{Na}}_{2}\mathrm{O} \to  {\mathrm{Na}}_{2}{\mathrm{ZnO}}_{2}$$;

$$ \mathrm{ZnO} + 2{\mathrm{NaOH}}_{\left(\mathrm{расплав}\right)} \to  {\mathrm{Na}}_{2}{\mathrm{ZnO}}_{2} + {\mathrm{H}}_{2}\mathrm{O}$$;
$$ \mathrm{ZnO} + 2\mathrm{NaOH} + {\mathrm{H}}_{2}\mathrm{O} \to  {\mathrm{Na}}_{2}[\mathrm{Zn} (\mathrm{OH}{)}_{4}]$$;

$$ \mathrm{ZnO} + 2\mathrm{HCl}  \to  {\mathrm{ZnCl}}_{2} + {\mathrm{H}}_{2}\mathrm{O}$$.

 

Помощь студентам в учёбе от Людмилы Фирмаль

Здравствуйте!

Я, Людмила Анатольевна Фирмаль, бывший преподаватель математического факультета Дальневосточного государственного физико-технического института со стажем работы более 17 лет. На данный момент занимаюсь онлайн обучением и помощью по любыми предметам. У меня своя команда грамотных, сильных бывших преподавателей ВУЗов. Мы справимся с любой поставленной перед нами работой технического и гуманитарного плана. И не важно: она по объёму на две формулы или огромная сложно структурированная на 125 страниц! Нам по силам всё, поэтому не стесняйтесь, присылайте.

Срок выполнения разный: возможно онлайн (сразу пишите и сразу помогаю), а если у Вас что-то сложное – то от двух до пяти дней.

Для качественного оформления работы обязательно нужны методические указания и, желательно, лекции. Также я провожу онлайн-занятия и занятия в аудитории для студентов, чтобы дать им более качественные знания.


Моё видео:



Вам нужно написать сообщение в Telegram . После этого я оценю Ваш заказ и укажу срок выполнения. Если условия Вас устроят, Вы оплатите, и преподаватель, который ответственен за заказ, начнёт выполнение и в согласованный срок или, возможно, раньше срока Вы получите файл заказа в личные сообщения.

Сколько может стоить заказ?

Стоимость заказа зависит от задания и требований Вашего учебного заведения. На цену влияют: сложность, количество заданий и срок выполнения. Поэтому для оценки стоимости заказа максимально качественно сфотографируйте или пришлите файл задания, при необходимости загружайте поясняющие фотографии лекций, файлы методичек, указывайте свой вариант.

Какой срок выполнения заказа?

Минимальный срок выполнения заказа составляет 2-4 дня, но помните, срочные задания оцениваются дороже.

Как оплатить заказ?

Сначала пришлите задание, я оценю, после вышлю Вам форму оплаты, в которой можно оплатить с баланса мобильного телефона, картой Visa и MasterCard, apple pay, google pay.

Какие гарантии и вы исправляете ошибки?

В течение 1 года с момента получения Вами заказа действует гарантия. В течении 1 года я и моя команда исправим любые ошибки в заказе.


Качественно сфотографируйте задание, или если у вас файлы, то прикрепите методички, лекции, примеры решения, и в сообщении напишите дополнительные пояснения, для того, чтобы я сразу поняла, что требуется и не уточняла у вас. Присланное качественное задание моментально изучается и оценивается.

Теперь напишите мне в Telegram или почту и прикрепите задания, методички и лекции с примерами решения, и укажите сроки выполнения. Я и моя команда изучим внимательно задание и сообщим цену.

Если цена Вас устроит, то я вышлю Вам форму оплаты, в которой можно оплатить с баланса мобильного телефона, картой Visa и MasterCard, apple pay, google pay.

Мы приступим к выполнению, соблюдая указанные сроки и требования. 80% заказов сдаются раньше срока.

После выполнения отправлю Вам заказ в чат, если у Вас будут вопросы по заказу – подробно объясню. Гарантия 1 год. В течении 1 года я и моя команда исправим любые ошибки в заказе.

















Можете смело обращаться к нам, мы вас не подведем. Ошибки бывают у всех, мы готовы дорабатывать бесплатно и в сжатые сроки, а если у вас появятся вопросы, готовы на них ответить.

В заключение хочу сказать: если Вы выберете меня для помощи на учебно-образовательном пути, у вас останутся только приятные впечатления от работы и от полученного результата!

Жду ваших заказов!

С уважением

Пользовательское соглашение

Политика конфиденциальности


Помогите!! ! что получится в реакциях? Pb3O4+HNO3+Fe(ClO4)2= Pb3O4+HNO3+KNO2= — Спрашивалка

АП

Анна Пестова

  • реакция

Сергей Пешехонцев

Оксид свинца Pb3O4 является смешанным оксидом свинца, действительная формула которого PbO2•2PbO. Поскольку свинец в оксиде Pb3O4 имеет две степени окисления +2 и +4, то оксид свинца Pb3O4 в окислительно-восстановительных реакциях может проявлять как окислительные, так и восстановительные свойства.

Pb3O4 + HNO3 + Fe(ClO4)2 =
Возможны два пути протекания данной реакции:
1. Когда окислителем является оксид свинца Pb3O4, точнее оксид свинца (IV) PbO2, содержащий свинец в высшей степени окисления +4, а восстановителем – катион железа Fe(2+).
2. Когда окислителем является перхлорат-ион ClO4(-), в котором хлор имеет высшую степень окисления, равную +7, а восстановителем — оксид свинца Pb3O4, точнее оксид свинца (II) PbO, имеющий степень окисления +2.
Азотная кислота, хоть и имеет азот в высшей степени окисления +5, но в данной реакции создает кислую среду реакции. Кроме того, при взаимодействии азотной кислоты с оксидом свинца Pb3O4 протекает реакция обмена между оксидом свинца (II) PbO и азотной кислотой.
PbO2•2PbO + 4HNO3 = 2Pb(NO3)2 + PbO2 + 2h3O или
Pb3O4 + 4HNO3 = 2Pb(NO3)2 + PbO2 + 2h3O
Чтобы выяснить, по какому пути будет протекать реакция, необходимо сравнить ЭДС двух реакций.
Е1 = φо [PbO2/Pb(2+)] — φо [Fe(3+)/Fe(2+)] = 1,455 – 0,77 = 0,685 В
Е2 = φо [ClO4(–)/Cl2] = 1,39 – 0,248 = 1,142 В
E2 > E1, следовательно реакция будет протекать по второму пути, то есть когда окислителем будет перхлорат-ион ClO4(-), а восстановителем — оксид свинца Pb3O4, точнее оксид свинца (II) PbO.
Расставим коэффициенты первого уравнения реакции методом электронно-ионного баланса.
Запишем сначала полуреакцию окисления для оксида свинца (II) PbO, а затем для всей молекулы оксида свинца Pb3O4.
В полуреакции окисления для молекулы оксида свинца Pb3O4 получилось 3 молекулы оксида свинца (IV) PbO2 – две образовались в результате окисления двух молекул оксида свинца (II) PbO (содержащихся в молекуле оксида свинца Pb3O4) и одна – не прореагировавшая ни с чем молекула оксида свинца (IV) PbO2 (содержащаяся в молекуле оксида свинца Pb3O4). Эта последняя молекула ни с чем не прореагировала, поскольку, имея свинец в высшей степени окисления, не может окисляться и в реакцию обмена с азотной кислотой не вступает.
PbO – 2e + h3O = PbO2 + 2Н (+) – окисление
Pb3O4 – 4е + 2Н2О = 3PbO2 + 4Н (+) | 14 — окисление
2ClO4(-) + 14e + 16H(+) = Cl2 + 8h3O | 4 – восстановление
4Pb3O4 + 28Н2О + 8ClO4(-) + 64H(+) = 12PbO2 + 56Н (+) + 4Cl2 + 32h3O
Приведем подобные
14Pb3O4 + 8ClO4(-) + 8H(+) = 12PbO2 + 4Cl2 + 4h3O
Получаем
Pb3O4 + 8HNO3 + 4Fe(ClO4)2 = 12PbO2 + 4Cl2↑ + 4Fe(NO3)2 + 4h3O

Pb3O4 + HNO3 + KNO2 =
Со второй реакцией все однозначно. Окислителем данной реакции является оксид свинца Pb3O4, точнее оксид свинца (IV) PbO2, содержащий свинец в высшей степени окисления +4, а восстановителем анион NO2(-), имеющий азот в промежуточной степени окисления +5.
Расставим коэффициенты первого уравнения реакции методом электронно-ионного баланса.
Запишем сначала полуреакцию восстановления для оксида свинца (IV) PbO2, а затем для всей молекулы оксида свинца Pb3O4.
В полуреакции восстановления для молекулы оксида свинца Pb3O4 получилось 3 катиона Pb(2+) – один образовался в результате восстановления оксида свинца (IV) PbO2, а два других – в результате реакции обмена между двумя молекулами оксида свинца (II) PbO (содержащихся в оксиде свинца Pb3O4) и азотной кислотой:
PbO + 2HNO3 = Pb(NO3)2 + h3O – реакция обмена
PbO2 + 4H(+) + 2e = Pb(2+) + 2h3O – полуреакция восстановления
Pb3O4 + 2e + 8H(+) = 3Pb(2+) + 4h3O | 1 – восстановление
NO2(-) – 2e + h3O = NO3(-) + 2H(+) | 1 – окисление
Суммарно
NO2(-) + Pb3O4 + 8H(+) + h3O = 3Pb(2+) + NO3(-) + 2H(+) + 4h3O
Приведем подобные
NO2(-) + Pb3O4 + 6H(+) = 3Pb(2+) + NO3(-) + 3h3O
Получаем
Pb3O4 + 6HNO3 + KNO2 = 3Pb(NO3)2 + KNO3 + 3h3O

Похожие вопросы

что получиться? Fe+h3+h3O=? ЧТО ПОЛУЧИТСЯ В ХОДЕ РЕАКЦИИ? И КАКИЕ КОЕФФИЦИЕНТЫ?

Помогите свершить реакции : Fe-Fe2O2-Fe(NO3)2-Fe(OH)2-Fe2O2

Помогите свершить реакции : Fe-Fe2O2-Fe(NO3)2-Fe(OH)2-Fe2O2. Помогите свершить реакции : Fe-Fe2O2-Fe(NO3)2-Fe(OH)2-Fe2O2

Закончить реакцию. . Помогите. . 1) Ba+HNO3(конц) 2) Mg+h3O 3) Fe+h3SO4(конц)

Посчитать степени окисления. HNO3 (Nh5)2CO3 N2O4 KNO2

Fe(OH)2+HNO3=Fe(NO3)3+NO+h3O

Записать уравнения реакций (СИУ, МУ, ПИУ) Fe2O3 + HNO3 ->Fe(OH)2 + HNO3

HNO3+KOH, HNO3+Cu(OH)2, HNO3+CuO, HNO3+Fe2O3, HNO3+K2SiO3 помогите пожалуйста!!!

C6H5COOH+ HNO3+h3SO4 = что получится? напишите ,пожалуйста ,реакцию!

Уравнени по химии (срочно) HNO3(роз) +NaO -> ?HNO3(роз) +KOH -> ?HNO3(роз) + Fe -> ?

Оксиды: классификация, получение и свойства

 

Оксиды — это сложные вещества, состоящие из атомов двух элементов, один из которых —  кислород со степенью окисления -2.  При этом кислород связан только с менее электроотрицательным элементом.

В зависимости от второго элемента оксиды проявляют разные химические свойства. В школьном курсе оксиды традиционно делят на солеобразующие и несолеобразующие. Некоторые оксиды относят к солеобразным (двойным).

Двойные оксиды — это некоторые оксиды , образованные элементом с разными степенями окисления.

Солеобразующие оксиды делят на основные, амфотерные и кислотные.

Основные оксиды — это оксиды, обладающие характерными основными свойствами. К ним относят оксиды, образованные атомами металлов со степень окисления +1 и +2.

Кислотные оксиды — это оксиды, характеризующиеся кислотными свойствами. К ним относят оксиды, образованные атомами металлов со степенью окисления +5, +6 и +7, а также атомами неметаллов.

Амфотерные оксиды — это оксиды, характеризующиеся и основными, и кислотными свойствами. Это оксиды металлов со степенью окисления +3 и +4, а также четыре оксида со степенью окисления +2: ZnO, PbO, SnO и BeO.

Несолеобразующие оксиды не проявляют характерных основных или кислотных свойств, им не соответствуют гидроксиды. К несолеобразующим относят четыре оксида: CO, NO, N2O и SiO.

Классификация оксидов

Тренировочные тесты по теме Классификация оксидов.

Получение оксидов

Общие способы получения оксидов:

1. Взаимодействие простых веществ с кислородом:

1.1. Окисление металлов: большинство металлов окисляются кислородом до оксидов с устойчивыми степенями окисления.

Например, алюминий взаимодействует с кислородом с образованием оксида:

4Al + 3O2 → 2Al2O3

Не взаимодействуют с кислородом золото, платина, палладий.

Натрий при окислении кислородом воздуха образует преимущественно пероксид Na2O2,

2Na + O2 → 2Na2O2

Калий, цезий, рубидий образуют преимущественно надпероксиды состава MeO2:

K + O2  →  KO2

Примечания: металлы с переменной степенью окисления окисляются кислородом воздуха, как правило, до промежуточной степени окисления (+3):

4Fe + 3O2 → 2Fe2O3

4Cr + 3O2 → 2Cr2O3

Железо также горит с образованием железной окалины — оксида железа (II, III):

3Fe + 2O2 → Fe3O4

 

1. 2. Окисление простых веществ-неметаллов.

Как правило, при окислении неметаллов образуется оксид неметалла с высшей степенью окисления, если кислород в избытке, или оксид неметалла с промежуточной степенью окисления, если кислород в недостатке.

Например, фосфор окисляется избытком кислорода до оксида фосфора (V), а под действием недостатка кислорода до оксида фосфора (III):

4P + 5O2(изб.) → 2P2O5

4P + 3O2(нед.) → 2P2O3

Но есть некоторые исключения.

Например, сера сгорает только до оксида серы (IV):

S + O2 → SO2

Оксид серы (VI) можно получить только окислением оксида серы (IV) в жестких условиях в присутствии катализатора:

2SO2 + O= 2SO3

Азот окисляется кислородом только при очень высокой температуре (около 2000оС), либо под действием электрического разряда, и только до оксида азота (II):

N2 + O2 = 2NO

Не окисляется кислородом фтор F2 (сам фтор окисляет кислород). Не взаимодействуют с кислородом прочие галогены (хлор Cl2, бром и др.), инертные газы (гелий He, неон, аргон, криптон).

 

2. Окисление сложных веществ (бинарных соединений): сульфидов, гидридов, фосфидов и т.д.

При окислении кислородом сложных веществ, состоящих, как правило, из двух элементов, образуется смесь оксидов этих элементов в устойчивых степенях окисления.

Например, при сжигании пирита FeS2 образуются  оксид железа (III) и оксид серы (IV):

4FeS2 + 11O2 → 2Fe2O3 + 8SO2

Сероводород горит с образованием оксида серы (IV)  при избытке кислорода и с образованием серы при недостатке кислорода:

2H2S + 3O2(изб.) → 2H2O + 2SO2

2H2S + O2(нед.) → 2H2O + 2S

А вот аммиак горит с образованием простого вещества N2, т. к. азот реагирует с кислородом только в жестких условиях:

4NH3 + 3O2 →2N2 + 6H2O

А вот в присутствии катализатора аммиак окисляется кислородом до оксида азота (II):

4NH3 + 5O2 → 4NO + 6H2O

 

3. Разложение гидроксидов. Оксиды можно получить также из гидроксидов — кислот или оснований. Некоторые гидроксиды неустойчивы, и самопроизвольную распадаются на оксид и воду; для разложения некоторых других (как правило, нерастворимых в воде) гидроксидов необходимо их нагревать (прокаливать).

гидроксид → оксид + вода

Самопроизвольно разлагаются в водном растворе угольная кислота, сернистая кислота, гидроксид аммония, гидроксиды серебра (I), меди (I):

H2CO3 → H2O + CO2

H2SO3 → H2O + SO2

NH4OH → NH3 + h3O

2AgOH → Ag2O + H2O

2CuOH → Cu2O + H2O

При нагревании разлагаются на оксиды большинство нерастворимых гидроксидов — кремниевая кислота, гидроксиды тяжелых металлов — гидроксид железа (III) и др. :

H2SiO3 → H2O + SiO2

2Fe(OH)3 → Fe2O3 + 3H2O

4. Еще один способ получения оксидов — разложение сложных соединений — солей.

Например, нерастворимые карбонаты и карбонат лития при нагревании разлагаются на оксиды:

Li2CO3 → CO2 + Li2O

CaCO3 →  CaO + CO2

Соли, образованные сильными кислотами-окислителями (нитраты, сульфаты, перхлораты и др.), при нагревании, как правило, разлагаются с с изменением степени окисления:

2Zn(NO3)2 → 2ZnO + 4NO2 + O2

Более подробно про разложение нитратов можно прочитать в статье Окислительно-восстановительные реакции.

Химические свойства оксидов

Значительная часть химических свойств оксидов описывается схемой взаимосвязи основных классов неорганических веществ.

Химические свойства основных оксидов

Подробно про химические свойства оксидов можно прочитать в соответствующих статьях:

Химические свойства основных оксидов.

Химические свойства кислотных оксидов.

Химические свойства амфотерных оксидов.

 

 

 

 

Понравилось это:

Нравится Загрузка…

Много свинца в воде? Может быть, виноват марганец — Polyprom

Марганец не является особо токсичным минералом. На самом деле людям нужно немного в своем рационе, чтобы оставаться здоровым.

Однако исследования в Вашингтонском университете в Сент-Луисе показали, что в сочетании с некоторыми другими химическими веществами природный марганец может привести к значительным изменениям в воде в свинцовых трубах. В зависимости от того, какие дезинфицирующие средства используются в воде, эти изменения могут иметь значительные — даже опасные — последствия.

Результаты были недавно опубликованы в журнале » Наука об окружающей среде и технологии» .

Исследование сосредоточено на уникальной форме свинца, PbO 2 или диоксида свинца (свинец в степени окисления плюс-4). Диоксид свинца имеет очень низкую растворимость в воде — он не легко растворяется в одной воде. Это также необычно по своей природе, в отличие от более знакомого PbCO 3 , карбоната свинца, который составляет чешуйки, которые имеют тенденцию формироваться на трубах.

«Вы не найдете PbO 2 в окружающей среде, потому что нет сильного окислителя», — сказал Дэниел Джаммар, профессор экологической инженерии Уолтера Э. Брауна в Инженерной школе Маккелви. «Но хорошие дезинфицирующие средства часто являются хорошими окислителями».

Хлор является отличным дезинфицирующим средством, поэтому он широко используется в питьевой воде в Америке и во всем мире. Он также является хорошим окислителем и способствует превращению карбоната свинца в диоксид свинца.

Оказывается, однако, что процесс не особенно быстр, и этот факт совпадает с некоторыми реальными системами, но, похоже, не с другими.

«Если вы посмотрите на систему, которая имеет свинцовые трубы и свободный хлор, то вы сделаете расчеты, вы ожидаете, что каждый из них будет иметь диоксид свинца на трубах», — сказал Джаммар. «Но мы этого не видим. Это заставляет нас задуматься: что-то еще влияет на то, заканчивается ли конкретная система диоксидом свинца на ее внутренней поверхности.

«Вот тут и появляется марганец».

В присутствии окислителей марганец может легко изменять степень окисления; если марганец вступает в контакт с хлором, он окисляется, превращаясь в оксид марганца. Как в компьютерных моделях, так и в экспериментах, имитирующих водопроводные трубы — в комплекте с искусственной водопроводной водой — лаборатория Гиаммара обнаружила, что оксид марганца действует как катализатор, увеличивая скорость превращения карбоната свинца в диоксид свинца на два порядка.

«Хлор по-прежнему является реагентом, определяющим конверсию свинца, но оксид марганца действует как катализатор, ускоряющий его», — сказал Джаммар.

Это исследование может помочь понять, как другие химические взаимодействия влияют на скорость превращения свинца. «Какие другие вещи, которые не являются свинцом, могут повлиять на эти показатели?» Спросил Джаммар. «Есть ли оксиды железа? Алюминий — это то, что мы тоже изучим».

Дальнейшие исследования в области понимания того, какие реакции влияют на скорость превращения свинца и каким-либо иным образом влияют на наличие свинца в воде, приведут не только к прорывам в лаборатории. Они будут иметь реальные последствия для здоровья.

Взять, к примеру, Вашингтон в 2000 году.

Управление по водоснабжению и канализации округа превратилось из дезинфицирующего средства, содержащего хлор, в менее сильное, называемое хлорамином, поскольку хлор создавал некоторые неприятные побочные продукты. Но было непредвиденное последствие.

«Когда водное управление переключило дезинфицирующее средство, диоксид свинца в трубном окалине больше не был стабильным», — сказал он. «Он быстро растворялся и генерировал высокие концентрации свинца в водопроводной воде».

События в DC заставили другие системы, использующие свободный хлор, начать задавать вопросы о том, должны ли они беспокоиться о диоксиде свинца, если они должны были перейти на хлорамин. Интересно, что многие системы наблюдают содержание диоксида свинца в весах на линиях технического обслуживания, но другие системы этого не делают. Различная концентрация марганца в общественных системах водоснабжения потенциально может объяснить эти различия.

«То, как вы собираетесь обрабатывать воду, зависит от источника и его состава, а также от вашей инфраструктуры», — сказал Джаммар. «Там не один размер подходит всем».

Это открытие было случайностью.

Лаборатория провела еще один эксперимент с искусственной водопроводной водой в свинцовых трубах и обработала ее хлором, чтобы увидеть, могут ли они создать диоксид свинца.

Они включали вещества, обычно встречающиеся в водопроводной воде: кальций, магний, натрий и хлорид. «Над проектом работала новая студентка, и вместо магния она добавила марганец», — сказал Джаммар.

Тогда все стало странным. «Вода была чистой, внезапно облачной и черной».

В течение нескольких недель было много осадков, но затем они стихли.

«Мы открыли трубы и посмотрели», — сказал Джаммар. «О, у нас есть диоксид свинца, который мы пытались сделать». Марганец просто ускорил процесс.

Уксусная кислота — Sciencemadness Wiki

Уксусная кислота (или этановая кислота ) представляет собой органическое соединение с химической формулой CH 3 COOH . Это бесцветная жидкость, которая в неразбавленном виде называется ледяной уксусной кислотой .

Уксус содержит приблизительно 4-8% уксусной кислоты по объему, и его характерный запах и вкус обусловлены уксусной кислотой. Хотя концентрированная уксусная кислота классифицируется как слабая кислота, она вызывает коррозию и может повредить кожу.

Содержание

  • 1 Свойства
    • 1.1 Физический
    • 1.2 Химическая
  • 2 Наличие
  • 3 Подготовка
  • 4 проекта
  • 5 Обращение
    • 5. 1 Безопасность
    • 5.2 Хранение
    • 5.3 Утилизация
  • 6 Каталожные номера
    • 6.1 Соответствующие темы Sciencemadness

Свойства

Физические свойства

Уксусная кислота представляет собой прозрачную бесцветную жидкость при комнатной температуре с немного более высокой вязкостью, чем вода, затвердевающая при температуре около 10 °C и кипящая при температуре от 117 до 118 °C. вкус. Уксусная кислота представляет собой полярный протонный растворитель, подобный этанолу и воде, но в отличие от воды уксусная кислота может растворять не только полярные соединения, такие как неорганические соли и сахара, но и неполярные соединения, такие как масла, и такие элементы, как сера и йод. Он легко смешивается с другими полярными и неполярными растворителями, такими как вода, хлороформ и гексан. С высшими алканами (начиная с октана) уксусная кислота больше не смешивается полностью, и ее смешиваемость продолжает снижаться с более длинными н-алканами.

Эта растворяющая способность и смешиваемость уксусной кислоты делают ее широко используемым промышленным химикатом, например, в качестве растворителя при производстве диметилтерефталата.

Химическая

Уксусная кислота вступает в химические реакции, типичные для карбоновой кислоты. При обработке стандартным основанием он превращается в ацетат металла и воду:

CH 3 COOH + MOH → CH 3 COOM + H 2

С супероснованиями (например, литийорганическими реагентами) он может быть подвергнут двойному депротонированию с получением LiCH 2 CO 2 Li. Восстановление уксусной кислоты дает этанол.

При нагревании выше 440 °C (824 °F) уксусная кислота разлагается с образованием метана и диоксида углерода или кетена и воды:

CH 3 COOH → CH 4 + CO 2
CH 3 COOH → CH 2 CO + H 2 O

Уксусная кислота вызывает умеренную коррозию металлов, включая магний, железо и цинк, образуя газообразный водород и ацетаты металлов.

Mg + 2 CH 3 COOH → Mg(CH 3 COO) 2 + H 2
Fe + 2 CH 3 COOH → Fe(CH 3 COO) 2 + H 2
Zn + 2 CH 3 COOH → Zn(CH 3 COO) 2 + H 2

При смешивании с достаточным количеством окислителя, обычно с перекисью водорода, уксусная кислота может непосредственно металлов (таких как медь) для получения ацетата, хотя для продолжения процесса потребуется добавить больше перекиси водорода. Ацетат меди можно получить таким образом. Ацетаты также можно получить из уксусной кислоты и соответствующего карбоната или гидроксида, как в популярной реакции с пищевой содой:

NaHCO 3 + CH 3 COOH → CH 3 COONa + CO 2 + H 2 O

Поскольку алюминиевые баки пассивируют кислотоупорную пленку, они образуют алюминиевую пленку для транспорта уксусной кислоты.

Уксусная кислота будет реагировать со спиртами в присутствии катализатора, такого как серная кислота, с образованием сложных эфиров:

CH 3 COOH + R-OH → CH 3 C(=O)O-R + H 2 O

Уксусная кислота реагирует с хлордегидратирующими агентами, такими как трихлорид/пентахлорид фосфора, тионилхлорид, сульфурилхлорид или фосген, с образованием ацетилхлорида:

3 CH 3 COOH + PCl 3 → CH 3 COCl + 3 HCl + H 3 PO 4
5 CH 3 COOH + PCl 5 → CH 3 COCl + 5 HCl + H 3 PO 4
2 CH 3 COOH + SOCl 2 → CH 3 COCl + HCl + SO 2
2 CH 3 COOH + SO 2 Cl 2 → CH 3 COCl + 2 HCl + SO 2
2 CH 3 COOH + COCl 2 → CH 3 COCl + 2 HCl + CO 2

Доступность.

и большое количество места, которое он требует, другие источники необходимы для эффективного использования в качестве реагента. Хорошим источником пищевой уксусной кислоты является уксусная эссенция, которая имеет концентрацию уксусной кислоты от 20 до 80%, в зависимости от марки, и ее можно найти во многих магазинах, хотя в некоторых странах ее стало трудно найти в последние годы.

Уксусная кислота также используется в фотографических стоп-ваннах, которые состоят из высокой концентрации забуференной уксусной кислоты, часто с включенным индикатором pH. Их можно удалить перегонкой. Ледяную уксусную кислоту также можно приобрести у онлайн-поставщиков по скромным ценам.

Уксусная кислота также продается в некоторых магазинах пчеловодов в различных концентрациях, таких как 60%, 80% и 100%.

Дистиллированный уксус не является дистиллированной уксусной кислотой. При производстве дистиллированного уксуса дистиллируются исходные ингредиенты, а НЕ конечный продукт. В продукте присутствуют значительные количества органических загрязнителей, которые, как можно наблюдать, разлагаются при нагревании. Ацетат натрия, приготовленный из пищевой соды и дистиллированного уксуса и вываренный досуха, имеет тенденцию быть коричневым, а не чисто белым.

Подготовка

Один из наиболее распространенных синтезов уксусной кислоты включает подкисление избытка ацетатной соли с получением уксусной кислоты. Этот несколько непрактичный метод требует добавления концентрированной серной кислоты к безводному ацетату кальция или ацетату натрия с последующей второй перегонкой над безводной гигроскопичной солью для удаления воды. Ацетатную соль в этом методе легко получить путем соединения домашнего уксуса с основанием и последующей кристаллизацией прореагировавшего раствора, но это очень утомительный процесс для получения больших количеств кислоты.

Проекты

  • Производство ацетатных эфиров, таких как метилацетат и этилацетат
  • Приготовление ацетата натрия или кальция
  • Удаление ржавчины
  • Стерилизация ульев
  • Кулинария (уксус, пищевой)

Обращение

Безопасность

Разбавленная уксусная кислота, она же уксус, раздражает нос и рот, а ледяная уксусная кислота вызывает коррозию. В остальном он не особенно токсичен.

Хранение

Хранение ледяной или концентрированной уксусной кислоты должно осуществляться в закрытых бутылях, вдали от металлов, подверженных ржавчине, лучше всего в кислотном шкафу.

Утилизация

Уксусную кислоту можно нейтрализовать основанием или карбонатом, таким как карбонат кальция. Концентрированную кислоту следует сначала разбавить перед нейтрализацией, чтобы предотвратить разбрызгивание.

Ссылки

  • http://en.wikipedia.org/wiki/Acetic_acid
  • Химические реагенты, их чистота и испытания E. Merck

Соответствующие темы Sciencemadness

  • Получение ледяной уксусной кислоты
  • Ледяная уксусная кислота…
  • Безрецептурная ледяная уксусная кислота нужна помощь
  • Получение уксусной кислоты?
  • Синтез уксусной кислоты. Легкая домашняя версия.
  • очищающая уксусная кислота
  • Белый уксус в уксусную кислоту
  • Очистка уксусной кислоты от уксуса
  • Ледяная уксусная кислота из уксусного ангидрида?
  • Синтез ледяной уксусной кислоты и серной кислоты
  • Уксусная кислота из ацетона
  • Ледяная уксусная кислота разбавление до 80%, от 99,9%
  • Дистилляция уксусной кислоты с энтрейнером
  • очистить уксусную кислоту

Спирт — Sciencemadness Wiki

Спирты — это органические соединения, которые содержат гидроксильную функциональную группу (-ОН), связанную с насыщенным атомом углерода.

Состав

  • 1 Типы спиртов
  • 2 Номенклатура
    • 2.1 Систематические названия
    • 2.2 Общие названия
  • 3 Физические и химические свойства
  • 4 приложения
  • 5 Подготовка
  • 6 реакций
    • 6.1 Окисление
    • 6.2 Этерификация
    • 6.3 Дегидратация
    • 6.4 Депротонирование
    • 6.5 Нуклеофильное замещение
  • 7 Безопасность
    • 7.1 Токсичность
  • 8 Каталожные номера
    • 8.1 Соответствующие темы Sciencemadness

Типы спиртов

Спирты подразделяются в зависимости от типа атома углерода, с которым связана гидроксильная функциональная группа:

  • Первичные спирты
    • Первичные спирты — это спирты, в которых атом углерода, связанный с гидроксильной функциональной группой, связан только с одним другим атомом углерода.
    • Первичные спирты сначала окисляются до альдегидов, затем до карбоновых кислот.
  • Вторичные спирты
    • Вторичные спирты – это спирты, в которых атом углерода, связанный с гидроксильной функциональной группой, связан с двумя другими атомами углерода.
    • Вторичные спирты окисляются до кетонов.
  • Третичные спирты
    • Третичные спирты – это спирты, в которых атом углерода, связанный с функциональной гидроксильной группой, связан с тремя другими атомами углерода.
    • Третичные спирты не окисляются.

Номенклатура

Систематические названия

Общие названия

Химическая формула Имя ИЮПАК Обычное имя
Одноатомные спирты
CH 3 ОН Метанол Древесный спирт
С 2 Н 5 ОХ Этанол Алкоголь
С 3 Н 7 ОХ 2-пропанол Медицинский спирт
С 4 Н 9 ОХ 1-бутанол Бутанол
С 5 Н 11 ОН 1-пентанол Спирт амиловый
С 16 Н 33 ОХ Гексадекан-1-ол Спирт цетиловый
Многоатомные спирты
С 2 Н 4 (ОН) 2 Этан-1,2-диол Этиленгликоль
C 3 H 6 (OH) 2 Пропан-1,2-диол Пропиленгликоль
C 3 H 5 (OH) 3 Пропан-1,2,3-триол Глицерин
С 4 Н 6 (ОН) 4 Бутан-1,2,3,4-тетраол Эритрит, Треитол
С 5 Н 7 (ОН) 5 Пентан-1,2,3,4,5-пентол Ксилит
С 6 Н 8 (ОН) 6 Гексан-1,2,3,4,5,6-гексол Маннит, сорбит
C 7 H 9 (OH) 7 Гептан-1,2,3,4,5,6,7-гептол Волемитол
Спирты алифатические ненасыщенные
С 3 Н 5 ОХ Проп-2-ен-1-ол Спирт аллиловый
С 10 Н 17 ОХ 3,7-Диметилокта-2,6-диен-1-ол Гераниол
С 3 Н 3 ОХ Проп-2-в-1-оле Пропаргиловый спирт
Спирты алициклические
C 6 H 6 (OH) 6 Циклогексан-1,2,3,4,5,6-гексол Инозитол
С 10 Н 19 ОХ 2-(2-пропил)-5-метилциклогексан-1-ол Ментол

Физические и химические свойства

Области применения

Подготовка

Метанол можно получить деструктивной перегонкой древесины, хотя в этом процессе также образуется формальдегид.

Этанол можно получить путем ферментации сахара.

Реакции

Окисление

Первичные спирты могут быть окислены либо до альдегидов, либо до карбоновых кислот, тогда как окисление вторичных спиртов обычно заканчивается на стадии кетона. Третичные спирты устойчивы к окислению.

Прямое окисление первичных спиртов до карбоновых кислот обычно протекает через соответствующий альдегид, который трансформируется через гидрат альдегида в результате реакции с водой, прежде чем он может быть дополнительно окислен до карбоновой кислоты.

Механизм окисления первичных спиртов в карбоновые кислоты через альдегиды и гидраты альдегидов

Реагенты, используемые для превращения первичных спиртов в альдегиды, обычно также подходят для окисления вторичных спиртов в кетоны. К ним относятся реактив Коллинза и периодинан Десс-Мартина. Непосредственное окисление первичных спиртов до карбоновых кислот можно осуществить с помощью перманганата калия или реактива Джонса.

Этерификация

Этерификация – это реакция спирта с кислотой, в результате которой образуется сложный эфир.

Подробнее на этой странице.

Дегидратация

Дегидратация – это процесс отделения всех молекул воды от спирта. Это делается с помощью дегидратирующего агента, который более гигроскопичен, чем спирт, одним из таких примеров являются щелочные металлы, обезвоженный хлорид кальция или безводный оксид кальция (негашеная известь).

Это также можно сделать с помощью молекулярных сит 3A.

Депротонирование

Депротонирование спиртов супероснованиями или активными металлами дает солеподобные соединения, называемые алкоксидами. Поскольку большинство спиртов являются более слабыми кислотами, чем вода, алкоксиды обычно полностью и необратимо гидролизуются в исходный спирт и гидроксид. Только некоторые алициклические кетоспирты, такие как родизоновая кислота, обладают достаточной кислотностью, чтобы сделать соответствующие алкоксиды устойчивыми в водном растворе.

Нуклеофильное замещение

Безопасность

Токсичность

Метанол ядовит, прием внутрь может привести к слепоте и смерти. Спирты с более длинной цепью, такие как этанол, н-пропанол и изопропанол, гораздо менее токсичны. Низкой токсичностью обладают также высшие спирты, такие как бутанол, пентанол и их изомеры.

За исключением этиленгликоля, большинство полиолов малотоксичны.

Одноатомные спирты могут всасываться через кожу.

Ссылки

  1. http://en.wikipedia.org/wiki/Alcohol

Соответствующие темы Sciencemadness

  • Синтез третичных спиртов с более длинной цепью
  • Изготовление этанола в домашних условиях.

неорганическая химия — Является ли PbO ионным соединением?

Спросил

Изменено 1 год, 3 месяца назад

Просмотрено 7 тысяч раз

$\begingroup$

В IIT-JAM 2018 был один вопрос: «Какой из следующих оксидов является ионным?» Я выбрал $\ce{PbO}$, но, согласно официальному ключу ответа, $\ce{PbO}$ не является ионным.

Хотя я знаю, что это различие между ионным и ковалентным не является четким. Но свинец — это металл, а кислород — неметалл. Так что, по моему мнению, он должен быть ионным. Я пытался найти в Интернете, но не смог узнать о $\ce{PbO}$, хотя $\ce{PbO2}$ определяется в соответствии с этой статьей между ионным и ковалентным. $\ce{PbO}$ должен быть более ионным, чем $\ce{PbO}$, так как степень окисления свинца ниже, он будет менее электроотрицательным.

Даже $\ce{PbCl2}$ считается ионным согласно многим книгам, где хлор менее электроотрицателен и намного больше по размеру по сравнению с кислородом, поэтому по крайней мере $\ce{PbO}$ должен иметь более ионный характер, чем $\ce{PbCl2}$ по правилу Фазана.

Так что, мне оспорить официальный ключ ответа? Если да, то на каком основании? Какую ссылку я должен отправить им?

Ионо-ковалентный характер связей металл–кислород в оксидах: сравнение экспериментальных и теоретических данных

  • неорганическая химия
  • металл
  • $\endgroup$

    3

    $\begingroup$

    Если вам нужна ссылка, чтобы оспорить вопрос, попробуйте страницу 248 Advanced Study Guide Chemistry. На этой странице есть таблица, в которой говорится, что PbO является «ионным». 92$ ]

    Итак, по шкале Полинга Э.Н. имеем Э.Н. из $\ce{Pb}$ = 1,87 (спасибо @Mithoron за исправление меня) и E.N. $\ce{O}$ = 3,44.

    Теперь, подставив значения, мы получим % ионного характера $\ce{PbO}$ на 33,75% , что, к сожалению для нас с вами (я тоже появился в этом году в ИИТ-ЖАМ и подал $\ ce{PbO}$ является ионным), указывает на то, что $\ce{PbO}$ имеет преимущественно ковалентный характер.

    $\endgroup$

    8

    $\begingroup$

    Одной из величайших ошибок во всей науке является широко распространенное представление о том, что существуют «ионные» и «ковалентные» связи, как если бы связь всегда была либо той, либо другой, и что мы можем отличить их по разнице электроотрицательности. .

    Ни то, ни другое не верно.

    • Связи всегда имеют смесь ионных и ковалентных вкладов, даже если они существуют между эквивалентными атомами, такими как $\ce{h3}$. Неполярные связи, как в молекулярном водороде, имеют небольшие противоположные ионные вклады, которые компенсируют друг друга.

    • Связи могут демонстрировать гораздо больший ионный характер, чем можно было бы ожидать, исходя из разницы электроотрицательности. Во многих случаях характер сильной ионной связи определяется молекулярно-орбитальной структурой, а не электроотрицательностью. Например, в дибориде магния бор образует макромолекулярные структуры с низкоэнергетическими вакантными орбиталями и, таким образом, имеет тенденцию становиться анионным (см. здесь).

    Мой ответ на этот вопрос: получить лучший учебник, когда он будет доступен.

    $\endgroup$

    Твой ответ

    Зарегистрируйтесь или войдите в систему

    Зарегистрируйтесь с помощью Google

    Зарегистрироваться через Facebook

    Зарегистрируйтесь, используя электронную почту и пароль

    Опубликовать как гость

    Электронная почта

    Требуется, но не отображается

    Опубликовать как гость

    Электронная почта

    Требуется, но не отображается

    Нажимая «Опубликовать свой ответ», вы соглашаетесь с нашими условиями обслуживания, политикой конфиденциальности и политикой использования файлов cookie

    .

    Определение, структура, свойства и использование

    Свинец находится в трех степенях окисления, и PbO 2 является одной из них. Он не встречается в природе в окружающей среде и поэтому создается в чрезвычайно окислительных условиях. Он используется в коммерческом смысле для производства взрывчатых веществ, спичек, красителей и т. Д.

    Таблица содержания

    1. .
    2. Production of PbO2
    3. Uses of PbO2
    4. Health Hazards of PbO2
    5. Things to Remember
    6. Sample Questions

    PbO 2 refers to lead IV окись. Это сильный окислитель, который представляет собой единую ковалентно связанную единицу. Он имеет массу 239,966 г/моль и плотность 9,38 г/см 3 .

    Подробнее:   Разница между электрофилом и нуклеофилом


    Что такое PbO

    2 ?
    • PbO 2 – это формула оксида свинца IV, также известного как диоксид свинца или безводная свинцовая кислота, или оксид свинца.
    • Степень окисления свинца в PbO 2 +4.
    • Сильный окислитель.
    • PbO 2 не существует в природе в нашей среде. Он производится в чрезвычайно окислительных условиях.

    Читайте также:


    Структура PbO

    2

    Оксид свинца IV имеет 2 акцептора водородной связи и 0 доноров водородной связи, поэтому соединение представляет собой структуру с одинарной ковалентной связью.

    Подробнее:


    Свойства PbO

    2
    • Оксид свинца IV представляет собой кристалл темно-коричневого или черного цвета.
    • Нерастворим в воде и спирте.
    • Растворим в кислотах, таких как разбавленная азотная кислота, щавелевая кислота, соляная кислота и т. д.
    • Масса моноизотопа оксида свинца IV составляет 239,966 г/моль.
    • The melting point of PbO 2 is 290 o C or 554 o C.
    • It has a density of 9.38 g/cm 3

    Read More:


    Production of PbO

    2

    Оксид свинца IV получают в промышленных масштабах различными способами. Некоторые из них:

    Окисление 

    При сурике (Pb 3 O 4 ) окисляется в щелочной суспензии в атмосфере хлора, затем выделяется оксид свинца IV (PbO 2 ).

    PB 3 O 4 + 4HNO 3 → PBO 2 + 2PB (№ 3 ) 2 + 2H 2 O

    Electrolisis

    2 o

    Electrolisis

    o

    . погружают в разбавленную серную кислоту и анодно поляризуют при электродном потенциале около +1,5 В при комнатной температуре, затем на чистом свинце получают оксид свинца IV.

    Читайте также:


    Использование PbO

    2

    Оксид свинца IV широко используется в промышленности для производства различных вещей, таких как:

    • красители, спички и заменители каучука;
    • стеклянная посуда. Стеклянная посуда, произведенная в прошлом, содержала огромное количество оксида свинца IV. Он был основным компонентом герметизирующих стекол при низких температурах обжига;
    • взрывчатые вещества;
    • отвердитель для полисульфидов;
    • свинцово-кислотные аккумуляторные батареи и активный материал положительного электрода;
    • медь гальваническая;
    • текстильная промышленность, в основном для окраски.

    Подробнее: Сульфат железа


    Опасность для здоровья PbO

    2
    • При нагревании оксида свинца IV образуются едкие, токсичные и раздражающие газы, которые при контакте с людьми обжигают их кожу и глаза.

    Подробнее:


    Что следует помнить

    • PbO — окислитель, находящийся в степени окисления +4.
    • Имеет одинарную ковалентно связанную структуру.
    • Кристаллы темно-коричневого или черного цвета, нерастворимые в воде и спирте, но растворимые в кислотах.
    • Не встречается в природе, но производится в промышленных масштабах путем окисления свинца и электролиза.
    • Используется в коммерческих целях для различных целей, например, для приготовления красителей, спичек, взрывчатых веществ, заменителей каучука, хранения аккумуляторов и т. д.
    • Хотя PbO 2 имеет множество применений, он опасен и токсичен для человека.

    Также проверьте:


    Примеры вопросов

    Вопросы. Что означает формула PbO 2 ?

    Ответ. PbO 2 относится к оксиду свинца IV. Он также известен как диоксид свинца, безводная свинцовая кислота или оксид свинца.

    Подробнее:

    Вопрос. Что происходит, когда PbO 2 греется?

    Ответ. При нагревании PbO 2 он разлагается. Реакция, которая происходит:

    PBO 2 → PB 12 O 19 → PB 12 O 17 → PB 3 O 4 → PB

    Прочтите: 79

    → PB

    .

    Вопросы. Как получают PbO 2 ?

    Ответ. Оксид свинца IV не встречается в природе, поэтому его производят в различных отраслях промышленности, например:

    • окисление свинцового сурика;
    • электролиз через медные и свинцовые электроды.

    Подробнее:

    Вопрос. Каково использование PbO 2 ?

    Ответ. Оксид свинца IV широко используется в промышленности для производства различных вещей, таких как:

    • красители, спички и заменители каучука;
    • стеклянная посуда. Стеклянная посуда, произведенная в прошлом, содержала огромное количество оксида свинца IV. Он был основным компонентом герметизирующих стекол при низких температурах обжига;
    • взрывчатые вещества;
    • отвердитель для полисульфидов;
    • Аккумуляторы свинцово-кислотные и активный материал положительного электрода;
    • медь гальваническая;
    • текстильная промышленность, в основном для окраски.

    Подробнее:

    Вопрос. Влияет ли контакт с PbO 2 на наше здоровье?

    Ответ. Да, оксид свинца IV является опасным элементом и токсичен при вдыхании или проглатывании. При нагревании оксида свинца IV образуются едкие, токсичные и раздражающие газы, которые при контакте с людьми обжигают кожу и глаза.

    Читайте также:

    Аномальные молярные массы: фактор Вант-Гоффа и коллигативные свойства

    Элементы группы 15: элементы P-блока, тенденции и свойства Химические реакции галоалканов и галоаренов

    Альдегиды, кетоны и карбоновые кислоты: использование, получение и структура

    Двуокись серы (SO2): структура, использование, получение и свойства

    Физические свойства аминов: структура амина и классификация

    Номенклатура спиртов, фенолов и эфиров: правила и примеры

    Межгалогенные соединения: типы, получение, свойства и применение

    Эмульсии: значение, типы, свойства и примеры

    Сбалансированные Химические уравнения: методы балансировки

    Энтальпия связи: определение, формула, примеры решения и видео

    Три состояния вещества: определение, свойства и примеры вопросов

    Дефект Шоттки: определение, характеристики, формула, вопросы предыдущего года

    Активированный комплекс: определение, формула, катализатор и факторы

    Оптическая изомерия: определение, возникновение, молекулы и примеры Солевой мостик

    Разница между изотопами и изобарами: примеры и использование

    Мальтоза: структура, свойства, использование и примеры вопросов

    Реакция нуклеофильного присоединения: различные механизмы, типы, примеры

    Открытие электрона: характеристики, эксперимент Дж. Дж. Томсона, открытие катодных лучей и решенные вопросы

    Неопределенность в измерении: научное обозначение, расчет, процентная формула и примеры

    Спектр водорода: серия, линейный спектр водорода, длина волны, примеры вопросов

    Принцип Ле Шателье: температура, давление, концентрация, катализатор

    Депрессия точки замерзания: формула, константа, расчет молярной массы, растворы

    Давление паров: закон Рауля, испарение, температура кипения

    Мицеллы: образование, структура, мыльное действие и примеры вопросов

    Фазовые превращения: типы, характеристики и примеры вопросов

    Восстановительный потенциал: определение, полуклетки и примеры вопросов

    Катализ: определение, классификация и примеры вопросов

    Комментарии



    Нет комментариев для показа

    ПОДПИСАТЬСЯ НА НАШИ НОВОСТИ

    УВЕДОМЛЕНИЯ КОЛЛЕДЖА

    УВЕДОМЛЕНИЯ ОБ ЭКЗАМЕНАХ

    ОБНОВЛЕНИЯ НОВОСТЕЙ

    Загрузите приложение Collegedunia по телефону

    © 2022 Collegedunia Web Pvt. Ltd. Все права защищены

    Окисление и восстановление

    Окисление и сокращение

    Окислительно-восстановительный Реакции Роль окисления Числа в окислительно-восстановительных реакциях Число окисления по сравнению с Истинный заряд ионов
    Окислители и Восстанавливающие агенты Конъюгат окисляющий Пары агент/восстановитель Родственник Сильные стороны металлов как восстановителей


    Окислительно-восстановительный Реакции

    Термин окисление первоначально использовался для описания реакции, в которых элемент соединяется с кислородом.

    Пример: Реакция между металлическим магнием и кислородом с образованием образование оксида магния включает окисление магния.

    Термин сокращение происходит от латинской основы, означающей «вернуть назад». Все, что ведет обратно к поэтому металлический магний включает восстановление.

    Реакция между оксидом магния и углеродом при 2000°С до из металлического магния и монооксида углерода является примером восстановление оксида магния до металлического магния.

    После открытия электрона химики убедились что окислительно-восстановительные реакции связаны с переносом электронов от одного атома к другому. С этой точки зрения, реакция между магнием и кислородом записывается следующим образом.

    2 Mg + O 2 2 [Mg 2+ ][O 2- ]

    В ходе этой реакции каждый атом магния теряет два электронов с образованием иона Mg 2+ .

    Mg Mg 2+ + 2 e

    Каждая молекула O 2 получает четыре электрона, образуя пара ионов O 2-.

    O 2 + 4 e 2 O 2-

    Поскольку электроны не создаются и не разрушается в результате химической реакции окисления и восстановления. связаны. Одно без другого невозможно, как показано на рисунке ниже.


    Роль окисления Числа в окислительно-восстановительных реакциях

    Химики со временем расширили идею окисления и сведение к реакциям, формально не связанным с переносом электронов.

    Рассмотрим следующую реакцию.

    CO( г ) + H 2 O( г ) CO 2 ( г ) + H 2 ( г )

    Как видно из рисунка ниже, общее количество электронов на валентной оболочке каждого атома остается постоянным в эта реакция.

    В этой реакции изменяется степень окисления этих атомы. Степень окисления углерода увеличивается с +2 до +4, в то время как степень окисления водорода уменьшается с +1 до 0.

    Поэтому лучше всего подходят окисление и восстановление определяется следующим образом. Окисление происходит при степень окисления атома становится больше. Сокращение происходит, когда степень окисления атома становится меньше.

     

    Интерактивный учебник по присвоению степеней окисления от

    требует Макромедиа Шоквейв

     


    Числа окисления по сравнению с Истинный заряд ионов

    Термины ионный и ковалентный описывают крайности континуума связи. есть какая-то ковалентная характер даже в самых ионных соединениях и наоборот.

    Полезно подумать о соединениях основной группы металлов, как если бы они содержали положительные и отрицательные ионы. химия оксида магния, например, легко понять если предположить, что MgO содержит Mg 2+ и О 2- ионы. Но никакие соединения не являются на 100% ионными. Есть экспериментальный доказательства, например, что истинный заряд магния и атомов кислорода в MgO +1,5 и -1,5.

    Степени окисления обеспечивают компромисс между мощной моделью окислительно-восстановительных реакций, основанных на предположении, что эти соединения содержат ионы и наши знания о том, что истинные заряд ионов в этих соединениях не так велик, как этот модель предсказывает. По определению, степень окисления атома заряд, который нес бы атом, если бы соединение было чисто ионный.

    Для активных металлов групп IA и IIA разница между степенью окисления атома металла и зарядом на этот атом достаточно мал, чтобы им можно было пренебречь. Металлы основной группы в Однако группы IIIA и IVA образуют соединения, которые имеют значительное количество ковалентного характера. Это заблуждение, ибо например, предположить, что бромид алюминия содержит Al 3+ ионы Br . На самом деле он существует как Al 2 Br 6 молекулы.

    Эта проблема становится еще более серьезной, когда мы обращаемся к химия переходных металлов. MnO, например, является ионным достаточно, чтобы считаться солью, содержащей Mn 2+ и O 2- ионы. Мн 2 О 7 , с другой стороны, представляет собой ковалентное соединение, которое кипит при комнатной температуре. Это Поэтому полезнее думать об этом соединении так, как если бы оно содержал марганец в степени окисления +7, а не Mn 7+ ионы.


    Окислители и Восстановители

    Рассмотрим роль каждого элемента в реакция, в которой определенный элемент приобретает или теряет электроны.

    Когда магний реагирует с кислородом, атомы магния отдают электронов на молекулы O 2 и тем самым уменьшить кислород. Таким образом, магний действует как восстановитель в эта реакция.

    2 мг + О 2 2 MgO
    восстановитель
    агент
         

    Молекулы O 2 , с другой стороны, получить электроны от атомов магния и тем самым окислить магний. Кислород, следовательно, окислитель .

    2 мг + О 2 2 MgO
      окислитель
    агент
       

    Окислители и восстановители, следовательно, могут быть определены как следует. Окислители получить электроны. Сокращение агенты теряют электроны.

    В таблице ниже указаны восстановитель и окислитель для некоторых реакций, обсуждаемых в этой сети страница. Сразу бросается в глаза одна тенденция: Металлы основной группы действуют как восстановители во всех своих химических реакциях.

    Типичные реакции металлов основной группы

    Реакция   Сокращение
    Агент
      Окислитель
    Агент
    2 Na + Cl 2 2 NaCl   На   Класс 2
    2 К + Н 2 2 КН   К   Н 2
    4 Li + O 2 2 Li 2 O   Ли   О 2
    2 Na + O 2 Na 2 O 2   На   О 2
    2 Na + 2 H 2 O 2 Na + + 2 OH + H 2   На   Н 2 О
    2 К + 2 NH 3 2 К + + 2 NH 2 + Н 2   К   НХ 3
    2 Mg + O 2 2 MgO   мг   О 2
    3 Мг + Н 2 Мг 3 Н 2   мг   Н 2
    Ca + 2 H 2 O Ca 2+ + 2 OH + H 2   Са   Н 2 О
    2 Al + 3 Br 2 Al 2 Br 6   Ал   Бр 2
    Мг + 2 Н + Мг 2+ + Н 2   мг   Н +
    Mg + H 2 O MgO + H 2   мг   Н 2 О


    Конъюгат окисляющий Пары агент/восстановитель

    Металлы действуют как восстановители в своих химических реакциях. Например, при нагревании меди над пламенем поверхность медленно чернеет, так как металлическая медь восстанавливает кислород в атмосфере с образованием оксида меди(II).

    Если мы выключим пламя и продуем H 2 газом над поверхность горячего металла, черный CuO, образовавшийся на поверхности металл медленно превращается обратно в металлическую медь. В ходе этой реакции CuO восстанавливается до металлической меди. Таким образом, Н 2 является восстановителем в этой реакции, а CuO действует как окислитель.

    Важной особенностью окислительно-восстановительных реакций может быть определяется путем изучения того, что происходит с медью в этой паре реакций. Первая реакция превращает металлическую медь в CuO, тем самым превращая восстановитель (Cu) в окислитель агент (CuO). Вторая реакция превращает окислитель (CuO) в восстановитель (Cu). Каждый восстановитель следовательно, связанный или связанный с сопряженным окислителем, и наоборот.

    Каждый раз, когда восстановитель теряет электроны, он образует окислитель, который мог бы получить электроны, если бы реакция была перевернутый.

    И наоборот, каждый раз, когда окислитель получает электроны, он образует восстановитель, который может потерять электроны, если реакция пошел в обратном направлении.

    Идея о том, что окислители и восстановители связаны, или спаренные, поэтому их называют сопряженными окисляющими агенты и восстановители. Сопряжение происходит от латинского основа, означающая «соединяться вместе». Поэтому используется для описания вещей, которые связаны или связаны, например, окисление агенты и восстановители.

    Все металлы основной группы являются восстановителями. Они, как правило, «сильные» восстановители. Активные металлы в группе ИА, например, лучше любых других элементов отдают электроны в периодической таблице.

    Тот факт, что активный металл, такой как натрий, является сильным восстановитель должен рассказать нам что-то об относительном сила Na 9Ион 0893 + в качестве окислителя. Если металлический натрий относительно хорошо отдает электроны, Na + ионы должны быть необычно плохо улавливают электроны. Если На является сильным восстановителем, ион Na + должен быть слабым окислитель.

    И наоборот, если O 2 имеет такое высокое сродство к электронов, что он необычайно хорошо принимает их от других элементы, он должен иметь возможность цепляться за эти электроны, как только он подбирает их. Другими словами, если O 2 сильный окислителем, то ион O 2- должен быть слабым Восстановитель.

    В целом взаимосвязь между сопряженным окислением и восстановители можно описать следующим образом. Каждый сильный восстановитель (например, Na) имеет слабый сопряженный окислитель (например, ион Na + ). Каждый сильный окислитель (такой как O 2 ) имеет слабую сопряженный восстанавливающий агент (такой как O 2- ион).


    Относительная сила металлов как восстановителей

    Мы можем определить относительную силу пары металлов как восстановители, определяя, происходит ли реакция, когда один одного из этих металлов смешивается с солью другого. Рассмотрим относительная прочность железа и алюминия, например. Ничего такого происходит, когда мы смешиваем порошкообразный металлический алюминий с оксидом железа (III). Однако если мы поместим эту смесь в тигель и получим реакция начинается с приложения небольшого количества тепла, бурная реакция происходит с образованием оксида алюминия и расплавленного металлического железа.

    2 Al( s ) + Fe 2 O 3 ( s ) Al 2 O 3 ( s ) + 2 Fe( l )

    Присвоив степени окисления, мы можем определить окисление и восстановление половин реакции.

    Алюминий окисляется до Al 2 O 3 в этом реакции, а это значит, что Fe 2 O 3 должно быть окислитель. И наоборот, Fe 2 О 3 есть восстанавливается до металлического железа, а это означает, что алюминий должен быть Восстановитель. Потому что восстановитель всегда превращается в сопряженный с ним окислитель в окислительно-восстановительной реакция, продукты этой реакции включают новый окислитель агент (Al 2 O 3 ) и новый восстановитель (Фе).

    Поскольку реакция идет в этом направлении, кажется разумно предположить, что исходные материалы содержат более сильный восстановитель и более сильный окислитель.

    Другими словами, если алюминий восстанавливает Fe 2 O 3 для формирования Al 2 O 3 и металлического железа, алюминий должен быть более сильным восстановителем, чем железо.

    Из того факта, что алюминий не может восстанавливать хлорид натрия с образованием металлического натрия, который исходные материалы в этой реакции более слабый окислитель и более слабый Восстановитель.

    Мы можем проверить эту гипотезу, спросив: что происходит, когда мы попробовать запустить реакцию в обратном направлении? (Натрий достаточно прочный металл, чтобы восстановить соль алюминия до алюминия металл?) При проведении этой реакции мы обнаруживаем, что металлический натрий может, фактически восстанавливают хлорид алюминия до металлического алюминия и натрия хлорида, когда реакцию проводят при температурах, достаточно высоких для расплавить реагенты.

    3 Na( l ) + AlCl 3 ( l ) 3 NaCl( l ) + Al( l )

    Если натрий достаточно силен, чтобы восстановить Al 3+ соли к металлическому алюминию, а алюминий достаточно прочен, чтобы восстановить солей Fe 3+ к металлическому железу, относительная сила эти восстановители можно резюмировать следующим образом.

    Na > Al > Fe

    Практическая задача 4:

    Использование следующие уравнения для определения относительного сильные стороны натрия, магния, алюминия и кальция металл как восстановитель.

    2 Na + MgCl 2 2 NaCl + Mg
    Al + MgBr 2
    Ca + MgI 2 CaI 2 + Mg
    Ca + 2 NaCl

    Нажмите здесь, чтобы проверить ваш ответ на практическое задание 4

    Samacheer Kalvi 12th Chemistry Solutions Chapter 3 p-Block Elements – II – Samacheer Kalvi

    Учащиеся могут загрузить главу 3 по химии p-Block Elements – II, вопросы и ответы, примечания Pdf, Samacheer Kalvi 12th Chemistry Solutions Guide Pdf поможет вам пересмотреть полную новую программу Совета штата Тамилнаду и получить больше баллов на экзаменах.

    Samacheer Kalvi 12th Chemistry p-Block Elements – II Оценка учебника

    I. Выберите правильный ответ.

    Вопрос 1.
    В каком из следующих случаев не используется NH 3 ?
    (а) Реактив Несслера
    (б) Реактив для анализа основного радикала IV группы
    (в) Реактив для анализа основного радикала III группы
    (г) Реактив Толлена
    Ответ:
    (а) Реактив Несслера

    Вопрос 2.
    Который час по азоту?
    (a) наименее электроотрицательный элемент
    (b) имеет меньшую энтальпию ионизации, чем кислород
    (c) доступные d-орбитали
    (d) способность образовывать связи pπ – pπ сами с собой
    Ответ:
    (d) способность образовывать pπ – pπ связывается сам с собой

    Вопрос 3.
    Элемент относится к 15 группе и 3 периоду таблицы Менделеева, его электронная конфигурация будет …………
    (a) 1s 2 2s 2 2p 4
    (b 1S 2 2S 2 2P 3
    (C) 1S 2 2S 2 2P 6 3S 2 3P 6 3S 2 3P 2 3S 2 3P 2 3S 2 3P 2 3 2 3P 2 3 2 3P 6 3 2 3P 2 3 2 3P 6 3S 6 3s 2 3p 3
    Ответ:
    (d) 1s 2 9Вопрос 4 A и B соответственно …………
    (a) P 4 (красный) и PH 3
    (b) P 4 (белый) и PH 3
    (c) S 8 и H 2 S
    (d) P 4 (белый) и H 2 S
    Ответ:
    (b) P 4 (белый) и PH 3

    Вопрос 5.
    В тесте на коричневое кольцо коричневый цвет кольца обусловлен …………
    (a) смесью NO и NO 2
    (b) Сульфат нитрозо железа
    (c) Нитрат железа
    (d) Нитрат железа
    Ответ:
    (b) Сульфат нитрозо железа

    Вопрос 6.
    При гидролизе PCl 3 дает …………
    (a ) H 3 PO 3
    (b) PH 3
    (c) H 3 PO 4
    (d) Пул
    Ответ:
    (a) H 3 PO 3

    Вопрос 7.
    P 4 O 6 Реагирует с холодной водой, чтобы дать …………
    . (a) H 3 PO 3
    (b) H 4 P 2 O 7
    (C) HPO 3
    (D) H 3 PO 4
    Ответ: D) H 3 PO 40009
    .
    (a) H 3 PO 3

    Вопрос 8.
    Основность пирофосфорной кислоты ( H 4 P 2 O 5 ) is …………
    (a) 4
    (b) 2
    (c) 3
    (d) 5
    Ответ:
    (b) 2

    Вопрос
    Молярность данного раствора ортофосфорной кислоты составляет 2М. его нормальность равна …………
    (а) 6N
    (б) 4N
    (в) 2N
    (г) ни один из этих
    Ответ:
    (а) 6N

    Вопрос 10.
    Утверждение – энергия диссоциации связи фтора больше, чем газообразного хлора
    Причина – хлор имеет большее электронное отталкивание, чем фтор
    (а) И утверждение, и разум истинны, и разум есть правильное объяснение утверждения.
    (b) И утверждение, и разум верны, но разум не является правильным объяснением утверждения.
    (c) Утверждение истинно, но причина ложна.
    (d) И утверждение, и причина ложны.
    Ответ:
    (d) И утверждение, и причина ложны. Обратное верно.

    Вопрос 11.
    Какой из перечисленных ниже окислителей является самым сильным?
    (а) Кл 2
    (б) F 2
    (c) Br 2
    (d) I 2
    Ответ:
    (b) F 2

    Вопрос 12.
    Правильный порядок термической устойчивости галогеноводорода… …
    (a) HI > HBr > HCl > HF
    (b) HF > HCl > HBr > HI
    (c) HCl > HF > HBr > HI
    (d) HI > HCl > HF > HBr
    Ответ:
    ( б) HF > HCl > HBr > HI

    Вопрос 13.
    Какое из следующих соединений не образуется?
    (a) xeof 4
    (b) XEO 3
    (c) XEF 2
    (D) NEF 2
    Ответ:
    (D) NEF 2

    ВОПРОС 14.
    Наиболее легко сжижаемый газ …………
    (а) Ar
    (б) Ne
    (в) He
    (г) Kr
    Ответ:
    (в) He

    Вопрос 15.
    XeF 6 при полном гидролизе дает …………
    (а) XeOF 4
    (б) XeO 2 F 4
    (в) XeO 3
    (г) XeO 9{ 2- }\)

    Вопрос 17.
    Какая из перечисленных кислот является самой сильной из всех?
    (a) HI
    (b) HF
    (c) HBr
    (d) HCl
    Ответ:
    (a) HI

    молекулы галогена?
    (а) Br 2 > I 2 > F 2 > Cl 2
    (b) F 2 > Cl 2 > > 90 9 2 0009
    (c) I 2 > BR 2 > CL 2 > F 2
    (D) CL 2 > BR 2 > F 2 > I 2
    Ответ:
    (г) Cl 2 > Br 2 > F 2 > I 2

    Вопрос 19. < HCIO < HClO 3 < HClO 4
    (б) HClO 4 2 3
    (C) HCLO 3 4 2 (D) HCIO 2 3 9 2 3 2 3 2 (D) 2 (D) 4
    Ответ:
    (г) HCIO < HClO 2 < HClO 3 < HClO 4

    Вопрос 20. а) CU(№ 3 ) 2 , НЕТ и НЕТ 2
    (b) Cu (№ 3 ) 2 и N 2 O
    (c) Cu (№ 3 ) 2 и № 2
    (D) CU (№ 3 ) 2 и NO
    Ответ:
    (c) CU(NO 3 ) 2 и NO 2

    II. Ответьте на следующие вопросы:

    Вопрос 1.
    Что такое эффект инертной пары?
    Ответ:
    По мере продвижения вниз по группе в элементах p-блока электроны ns² становятся инертными и не участвуют в химическом соединении.
    В химическом соединении принимают участие только np-электроны. Это называется эффектом инертной пары.

    Вопрос 2.
    Халькогены относятся к р-блоку. Дайте повод.
    Ответ:
    Халькогены — рудообразующие элементы. Большинство руд представляют собой оксиды и сульфиды, поэтому кислород, сера и другие элементы 16 группы называются халькогенами. В O, S, Se, Te и Po последний электрон переходит на р-орбиталь. Следовательно, халькогены относятся к р-блоку.

    Вопрос 3.
    Объясните, почему фтор всегда проявляет степень окисления -1?
    Ответ:

    • Поскольку фтор является наиболее электроотрицательным элементом, он проявляет только отрицательное состояние -1.
    • Из-за отсутствия d-орбиталей фтор не проявляет положительных степеней окисления.

    Вопрос 4.
    Ниже укажите степень окисления галогена.

    1. ИЗ 2
    2. О 2 Ж 2
    3. Класс 2 О 3
    4. И 2 О 4

    Ответ:
    1. OF 2
    +2 + 2(x) = 0
    +2 = -2x
    2x = -2
    x = -1

    2. O 2 F 8 2 2(+1) + 2x = 0
    2x = – 2
    x = – 1

    3. Cl 2 O 3
    2(x) + 3(-2) =0
    2x = +6
    x = +3

    4. I 2 O 4
    2(x) + 4(-2) =0
    2x = +8
    x = +4

    Вопрос 5.
    Что такое интергалогены соединения? Приведите примеры.
    Ответ:
    Каждый галоген соединяется с другими галогенами с образованием ряда соединений, называемых межгалогенными соединениями.
    (ex) ClF, BrF 3 , IF 5 , If 7

    Вопрос 6.
    Почему фтор более активен, чем другие галогены?
    Ответ:
    Фтор является наиболее активным элементом среди галогенов. Это связано с минимальным значением энергии диссоциации связи F – F. Следовательно, фтор более реакционноспособен, чем другие галогены.

    Вопрос 7.
    Укажите применение гелия.
    Ответ:

    1. Смесь гелия и кислорода используется водолазами вместо воздушно-кислородной смеси. Это представляет болезненное опасное состояние, называемое изгибами.
    2. Гелий используется для создания инертной атмосферы при электродуговой сварке металлов.
    3. Гелий имеет самую низкую температуру кипения, поэтому используется в криогенике (наука о низких температурах)
    4. Гелий гораздо менее плотный, чем воздух, поэтому его используют для наполнения воздушных шаров.

    Вопрос 8.
    Что представляет собой гибридизация йода в IF 7 ? Дайте его структуру.
    Ответ:
    Гибридизация йода в IF 7 is sp 3 d 3 Структура IF 7 пентагонально-бипирамидальная.

    Вопрос 9.
    Приведите уравнение реакции хлора с холодным NaOH и горячим NaOH.
    Ответ:
    1. Реакция хлора с холодным NaOH:
    Cl 2 + H 2 O → HCl + HOCl
    HCl + NaOH → NaCl + H 2 O
    HOCl + NaOH → NaOCl + H 2 O
    Общая реакция

    Хлор реагирует с холодным NaOH с образованием хлорида натрия и гипохлорита натрия.

    2. Реакция хлора с горячим NaOH:
    Cl 2 + H 2 O → HCl + HOCl
    HCl + NaOH → NaCl + H 2 O
    HOCl + NaOH → NaOCl + H

    9 2 O


    3NaOCl → NaClO 3 + 2NaCl
    Общая реакция

    Хлор реагирует с горячим NaOH с образованием хлората натрия и хлорида натрия.

    Вопрос 10.
    Как вы будете готовить хлор в лаборатории?
    Ответ:
    1. Хлор получают действием серной кислоты на хлориды в присутствии двуокиси марганца.
    4NACL + MNO 2 + 4H 2 SO 4 → CL 2 + MNCL 2 + 4NAHSO 4 + 2H 2 O

    2. Он также может быть подготовлен путем оксидирования Hydrochlicic Ascloric Actidric Actidric Actidric Actidric Actidric Actidric Actidric Cloisric. с использованием различных окислителей, таких как диоксид марганца, диоксид свинца, перманганат калия или дихромат.
    PBO 2 + 4HCl → PBCL 2 + 2H 2 O + CL 2
    MNO 2 + 4HCL → MNCL 2 + 2H 2 O + CL 2 9217 2009 2KMNO 2 O + CL 2 9000 2009 2K 40009. + 16hcl → 2Kcl + 2mncl + 8h 2 O + 5Cl 2
    K 2 CR 2 O 7 + 14HCl → 2Kcl + 2CRCL 3 + 7H 2 O + 3Cl + 2CR

    3. При обработке хлорки минеральными кислотами выделяется хлор
    CAOCL 2 + 2HCl → CACL 2 + H 2 O + CL 2
    CAOCL 2 + H 2 SO 4 → CASO 4 + H 2 O + CL 2

    Вопрос 11.
    Укажите применение серной кислоты.
    Ответ:

    1. Серная кислота используется в производстве удобрений, сульфата аммония и суперфосфатов, а также других химических веществ, таких как соляная кислота, азотная кислота и т. д.
    2. Используется в качестве осушителя, а также при приготовлении пигментов, взрывчатых веществ и т. д.

    Вопрос 12.
    Обоснуйте, что серная кислота является дегидратирующим агентом.
    Ответ:
    Серная кислота хорошо растворяется в воде и имеет сильное сродство к воде, поэтому ее можно использовать в качестве дегидратирующего агента. При растворении в воде образует моно(H2SO4.H2O) и ди(H2SO4.2H2O) гидраты, реакция экзотермическая. Свойство дегидратации можно также проиллюстрировать его реакцией с органическими соединениями, такими как сахар, щавелевая кислота и муравьиная кислота.

    Вопрос 13.
    Напишите причину аномального поведения Азота.
    Ответ:
    Причина аномального поведения азота

    • Маленький размер
    •  Высокая электроотрицательность
    • Отсутствие d-орбиталей в валентной оболочке
    • Химически инертен из-за высокой энергии связи

    Вопрос 14.
    Напишите молекулярную формулу и структурную формулу следующих молекул.
    (а) Азотная кислота
    (b) пятиокись азота
    (c) фосфорная кислота
    (d) фосфин
    Ответ:

    Вопрос 15.
    Укажите применение аргона.
    Ответ:

    • Аргон предотвращает окисление горячей нити накаливания и продлевает срок службы ламп накаливания.
    • Аргон используется в радиоклапанах и трубках.

    Вопрос 16.
    Напишите электронную конфигурацию валентной оболочки элементов группы 15.
    Ответ:
    Общая электронная конфигурация элементов группы 15: ns 2 нп 3 .

    1. Азот – [He] 2s 2 2p 3
    2. Фосфор – [Ne] 3s 2 3p 3
    3. Мышьяк – [Ar] 3d 10 4s 2 4p 3
    4. Сурьма – [Kr] 4d 10 5s 2 5p 3
    5. Висмут – [Ne] 4f 14 5s 10 6s 2 6p 3

    Вопрос 17.
    Приведите два уравнения, иллюстрирующие химическое поведение фосфина.
    Ответ:
    1. Фосфин реагирует с галогенами с образованием пентагалогенидов фосфора.
    PH 3 + 4Cl 2 → PCl 5 + 3HCl

    2. Фосфин образует координационное соединение с кислотами Льюиса, такими как трихлорид бора.

    3. Фосфин осаждает некоторые металлы из растворов их солей.
    3AgNO 3 + PH 3 → Ag 3 P + 3HNO 3

    Вопрос 18.
    Приведите реакцию между азотной кислотой и основным оксидом.
    Ответ:
    Азотная кислота реагирует с основаниями и основными оксидами с образованием солей и воды.

    1. ZnO + 2HNO 3 → Zn(NO 3 ) 2 + H 2 O
    2. 3FeO + 10HNO 3 → 3Fe(NO 3 ) 3 + NO + 5H 2 O

    Вопрос 19.
    Что происходит при нагревании PCl 5 ?
    Ответ:
    При нагревании пентахлорид фосфора разлагается на трихлорид фосфора и хлор.
    PCl 5 \(\underrightarrow { \triangle }\) PCl 3 + Cl 2

    Вопрос 20.
    Укажите причину, по которой HF является слабой кислотой, тогда как бинарные кислоты всех остальных галогенов являются сильными кислоты.
    Ответ:
    Галогениды водорода хорошо растворяются в воде благодаря ионизации.
    X + H 2 O → H 3 O + + X
    ( X = F, Cl, Br или I )

    Растворы галогеноводородов поэтому кислые и известны как галогеноводородные кислоты. Соляная, бромистоводородная и йодистоводородная кислоты почти полностью ионизированы и поэтому являются сильными кислотами, а HF — слабой кислотой. Для ВЧ, 9{–}\)

При высокой концентрации важно равновесие, связанное с удалением ионов фтора. Поскольку он влияет на диссоциацию фтористого водорода, следовательно, это слабая кислота.

Вопрос 21.
Определите степень окисления кислорода в фторноватистой кислоте – HOF.
Ответ:
В случае связи O – F HOF, фтор является самым электроотрицательным элементом. Значит, его степень окисления равна -1. Таким образом, степень окисления O равна +1. Точно так же в случае связи O – H это HOF. O более электроотрицателен, чем H. Таким образом, его степень окисления равна -1, а степень окисления H равна +1. Таким образом, чистое окисление кислорода равно – 1 + 1 = 0,9.2117

Вопрос 22.
Какой тип гибридизации встречается у

  1. BrF 5
  2. БрФ 3

Ответ:
1. BrF 5
BrF 5 относится к типу AX 5 . Поэтому он имеет гибридизацию sp 3 d 2 . Следовательно, молекула BrF 5 имеет форму квадратной пирамиды.

2. BrF 3
BrF 3 относится к типу AX 3 . Поэтому он имеет sp 3 г гибридизация. Следовательно, молекула BrF 3 имеет Т-образную форму.

Вопрос 23.
Закончите следующие реакции.
Ответ:

Samacheer Kalvi 12th Chemistry p-Block Elements – II Оцените себя

Вопрос 1.
Напишите продукты, образующиеся при реакции азотной кислоты (как разбавленной, так и концентрированной) с цинком.
Ответ:

Samacheer Kalvi 12th Chemistry p-Block Elements – II Дополнительные вопросы

Samacheer Kalvi 12th Chemistry p-Block Elements – II 1 балл Вопросы и ответы

I. Выберите лучший ответ.

Вопрос 1.
Около 78% земной атмосферы содержит,…………
(а) Р
(б) Ас
(в) N
(г) Bi
Ответ:
(в) N

Вопрос 2
Что из следующего не является пниктогеном?
(a) Азот
(b) Кислород
(c) Фосфор
(d) Сурьма
Ответ:
(b) Кислород

Вопрос 3.
Что из следующего показывает изотопы?
(a) Азот
(b) Мышьяк
(c) Сурьма
(d) Висмут
Ответ:
(a) Азот


(a) простая перегонка
(b) фракционная перегонка
(c) сублимация
(d) перегонка при пониженном давлении
Ответ:
(b) фракционная перегонка

Вопрос 5.
Порядок связи в молекуле азота ……… …
(а) 1
(б) 2
(в) 3
(г) 0
Ответ:
(в) 3

б) Благородный
(в) Более реактивный
(г) Менее реактивный
Ответ:
(а) Инертный

Вопрос 7.
Что из следующего используется в криохирургии?
(a) Liq N 2
(b) Liq NH 3
(c) Liq Na
(d) Liq H 2
Ответ:
(a) Liq N 2

  • Вопрос 8.
    Диэлектрическая проницаемость аммиака равна (К) …………
    (а) 10 -30
    (б) 10 -14
    (в) 10 30
    (г) 14
    Ответ:
    (а) 10 -30

    Вопрос 9.
    При взаимодействии аммиака с раствором медного купороса с образованием комплекса цвет комплекса …………
    (а) фиолетовый
    (б) темно-синий
    (c) синий
    (d) красный
    Ответ:
    (b) темно-синий

    Вопрос 10.
    Н – N – Н валентный угол в NH 3 равен …………
    (а) 109° 28′
    (б) 107° 28′
    (в) 104°
    (г) 107°
    Ответ:
    (d) 107°

    Вопрос 11.
    Форма аммиака …………
    (a) Плоская
    (b) Квадратная плоская
    (c) Пирамидальная
    (d) Квадратная пирамидальная
    Ответ:
    (c) Пирамидальная

    Вопрос 12.
    Азотная кислота, полученная в больших масштабах с использованием …………
    (a) Способ Оствальда
    (b) Способ Габера
    (c) Контактный процесс
    (d) Способ Дикона
    Ответ:
    (а) Процесс Оствальда

    Вопрос 13.
    Бензол вступает в реакцию нитрования с образованием нитробензола в этой реакции происходит за счет образования …………
    (а) Иона гидроксония
    (б) Гидрид-иона
    (в) Ион нитрония
    (г) Ион нитрония
    Ответ:
    (в) Ион нитрония

    Вопрос 14. ) +3
    (в) +4
    (г) +5
    Ответ:
    (г) +5

    Вопрос 15.
    Соединение, используемое в фотографии …………
    (a) AgNO 3
    (b) AgBr
    (c) AgCl
    (d) AgI
    Ответ:
    (a) AgNO 3

    8 3

    Вопрос 16.
    Нитрат натрия
    (а) Фотография
    (б) Огнестрельное оружие
    (в) Королевская вода Сосургерв …………
    (г) Криохирургия
    Ответ:
    (б) Огнестрельное оружие

    92 2 Вопрос Цвет полуторного оксида азота …………
    (а) бесцветный
    (б) коричневый
    (в) синий
    (г) Красный
    Ответ:
    (в) Синий

    Вопрос 18.
    Белый фосфор еще называют…………
    (а) Красный фосфор
    (б) Черный фосфор
    (в) Алый фосфор
    (г) Желтый фосфор
    Ответ:
    (г) Желтый фосфор

    Вопрос 19.
    Белый (желтый) фосфор светится в темноте в результате окисления, которое называется …………
    (а) фосфоресценция
    (б) фосфор
    (в) Флуоресценция
    (г) Лиминосценция
    Ответ:
    (а) фосфоресценция

    Вопрос 20.
    Желтый фосфор реагирует со щелочью при кипячении в инертной атмосфере с выделением …………
    (а) Фосфорная кислота
    (б) Фосфорная кислота
    (в) Фосфин
    (г) Пирофосфорная кислота
    Ответ:
    (c) Фосфин

    Вопрос 21.
    Рассмотрим следующие утверждения.
    (i) фосфин – важнейший гидрид фосфора
    (ii) фосфин – ядовитый газ с запахом тухлых яиц.
    (iii) фосфин является сильным восстановителем

    Какое из приведенных выше утверждений верно?
    (a) (i) и (ii)
    (b) (ii) и (iii)
    (c) (i) и (iii)
    (d) (ii) только
    Ответ:
    (c) (i ) и (iii)

    Вопрос 22.
    При нагревании фосфина на воздухе он сгорает с образованием …………
    (a) Ортофосфорная кислота
    (b) Метафосфорная кислота
    (c) Пирофосфорная кислота
    (d) Триоксид фосфора
    Ответ:
    (b) Метафосфорная кислота

    Вопрос 23.
    Гибридизация P в фосфине …………
    (a) sp 3 d
    (b) sp 3 d 2
    (c) sp3d3
    (d) sp3
    …………
    (a) Фосфин + Ацетилен
    (b) H 3 PO 3 +H 3 PO 3
    (c) Карбид кальция + фосфид кальция + карбонат кальция
    (d) фосфат
    Ответ:
    (c) Карбид кальция + фосфид кальция

    Вопрос 25.
    Хальгогены также называются …………
    (а) Рудообразующие элементы
    (б) Элементы группы 16
    (в) Элементы группы 17
    (г) Оба (а) и (б)
    Ответ:
    (г) Оба (а) и (б)

    Вопрос 26.
    Элемент, присутствующий в вулканическом пепле, это …………
    (a) Кислород
    (b) Сера
    (c) Селен
    (d) Теллур
    Ответ:
    (b) Сера

    Вопрос
    Разложение хлората калия ускоряется в присутствии …………
    (а) MnO 2
    (з) Mn 3 O 4
    (C) MNSO 4
    (D) KMNO 4
    Ответ:
    (A) MNO 2

    Вопрос 28.
    Чистый Озон… (a) желтый газ
    (b) голубой газ
    (c) бледно-голубой газ
    (d) ярко-голубой газ
    Ответ:
    (c) бледно-голубой газ

    Вопрос 29.
    Форма озона …………
    ( а) V-образная форма
    (б) Линейная форма
    (в) изогнутая форма
    (г) сферическая форма
    Ответ:
    (в) изогнутая форма

    Вопрос 30.
    Скорость разложения озона резко падает в …………
    (а) кислой среде
    (б) щелочной среде
    (в) нейтральной среде
    (г) эфирной среде
    Ответ:
    ( б) щелочная среда

    Вопрос 31.
    Что из перечисленного используется в качестве топлива в ракетах?
    (a) Liq O 2
    (b) Liq CO 2
    (c) Liq N 2
    (d) Liq He – O 2

    Вопрос 32.
    Найдите кристаллическую аллотрофную форму серы?
    (a) γ – сера
    (b) λ – сера
    (c) α – сера
    (d) молоко серы
    Ответ:
    (c) α – сера

    Вопрос 33.
    Рассмотрим следующие утверждения
    ( и) α – сера является единственной термодинамически стабильной аллотрофной формой.
    (ii) При 140 °C моноклинная сера плавится с образованием подвижной бледно-желтой жидкости, называемой γ-сера
    (iii) Моноклинная сера стабильна при температуре 96°C-119°C и медленно превращается в λ-сера

    Какое из приведенных выше утверждений верно/неверно?
    (a) (i) только
    (b) (ii) только
    (c) (iii) только
    (d) (ii) и (iii)
    Ответ:
    (d) (ii) и (iii)

    Вопрос 34.
    Диоксид серы, во сколько раз тяжелее воздуха?
    (a) 2 раза
    (b) 2,5 раза
    (c) 2,2 раза
    (d) 2,3 раза
    Ответ:
    (c) 2,2 раза

    Вопрос 35.
    Какое из перечисленных веществ обладает временным отбеливающим действием?
    (a) хлор
    (b) SO 3
    (c) H 3 SO 4
    (D) SO 2
    Ответ:
    (D) SO 2

    ).
    Серная кислота может быть получена …………
    (a) Процесс Оствальда
    (b) Процесс со свинцовой камерой
    (c) Процесс Дикона
    (d) Процесс Габера
    Ответ:
    (b) Процесс со свинцовой камерой

    Вопрос 37.
    Кислота серная производится контактным способом, в контактном процессе используется катализатор …………
    (a) V 2 O 5
    (b) Ticl 4
    (c) Fe
    (D) MO
    Ответ:
    (c) V 2 O 5

    . Вопрос. 38.
    Бензол реагирует с серной кислотой с образованием …………
    (a) сульфат
    (b) сульфид
    (c) сульфокислота
    (d) сульфит
    Ответ:
    (c) сульфокислота

    Вопрос 39.
    Реагент, используемый для обнаружения сульфат-иона: …………
    (a) BaCl 2
    (b) BaSO 3
    (c) (CH,COO),Pb
    (d) оба (a) и (c)
    Ответ:
    (d) оба (a) и (c)

    Вопрос 40.
    Процесс Дикона используется для изготовления …………
    (а) кл. 41.
    Катализатор, используемый в процессе Дикона, представляет собой …………
    (a) CuCl 2
    (b) Cu 2 Cl 2
    (c) CuBr
    (d) Cu 2 BR 2
    Ответ:
    (b) CU 2 CL 2

    Вопрос 42.
    C 10 H 16 + 8C 12 \ (\ indrightRow {\ triangle} \ ) А. Определить А?
    (a) Метан
    (b) Этан
    (c) Уголь
    (d) Пропан
    Ответ:
    (c) Уголь

    Вопрос 43. (а) CaOCl
    (б) CaOCl 2
    (в) CaO
    (г) CaCl 2
    Ответ:
    (b) CaOCl 2

    Вопрос 44.
    Что из перечисленного используется для очистки питьевой воды?
    (a) SO 3
    (b) SO 2
    (c) BR 2 / H 2 O
    (D) CL 2
    Ответ:
    (D) Cl 2

    Вопрос 45.
    Какая из следующих кислот является слабой?
    (а) HF
    (б) HCl
    (в) HBr
    (г) HI
    Ответ:
    (а) HF

    Вопрос 46.
    Реагент не хранится в стеклянных бутылях?
    (a) HCI
    (b) HBr
    (c) HF
    (d) HI
    Ответ:
    (c) HF

    Вопрос 47.
    Более активным элементом является
    (a) Фтор
    (b) Хлор
    ( в) Бром
    (г) Йод
    Ответ:
    (а) Фтор

    Вопрос 48.
    Правильный порядок кислотности галогеноводородных кислот?
    (a) HF > HCl > HBr > HI
    (b) HCl > HF > HBr > HI
    (c) HBr > HCl > HF > HI
    (d) HI > HBr > HCl > HF
    Ответ:
    (d) HI > HBr > HCl > HF

    Вопрос 49.
    Рассмотрим следующие утверждения
    (i) В межгалогенных соединениях центральный атом будет меньшим один.
    (ii) Может образовываться только между двумя галогенами и не более чем двумя галогенами.
    (iii) Они являются сильными восстановителями.

    Какое из приведенных выше утверждений неверно?
    (a) (i) только
    (b) (ii) и (iii)
    (c) (i) и (iii)
    (d) (iii) только
    Ответ:
    (c) (i) и (iii)

    Вопрос 50.
    Форма CLF 3 …………
    (a) Линейная
    (b) Т-образная
    (c) Пирамидальная
    (d) Плоский квадрат
    Ответ:
    (b) T-образный

    Вопрос 51.
    Какое из следующих веществ более кислое?
    (a) hocl
    (b) HCIO 2
    (c) HCLO 3
    (D) HCLO 4
    Ответ:
    (D) HCLO 4

    Вопрос 52.
    , когда XFO 6 8 60009

    Вопрос 52.
    . реагирует с 2,5 м NaOH дает …………
    (a) Na 4 XEO 6
    (B) NA 2 XEO 3
    (C) XEO 2 F 2
    (D). ) XEO 3
    Ответ:
    (a) NA 4 XEO 6

    Вопрос 53.
    Форма XEF 6 IS …………
    (A) Octahedron
    (B) Dexorted Octahedronronronronronronronronron
    (в) Пирамидальный
    (г) Тетраэдр
    Ответ:
    (б) Искаженный октаэдр

    Вопрос 54.
    Что из перечисленного может проникать сквозь густой туман?
    (a) He
    (b) Ne
    (c) Kr
    (d) Rn
    Ответ:
    (c) Kr

    Вопрос 55.
    Найдите радиоактивный элемент?
    (а) He
    (б) Rn
    (в) Xe
    (г) Ar
    Ответ:
    (б) Rn

    II. Заполните пропуски:

    1. Одиннадцатый по распространенности элемент ………….
    2. …………. является основным газом атмосферы.
    3. Азот химически ………….
    4. …………. процесс синтеза аммиака.
    5. …………. используется для производства цианамида кальция
    6. …………. представляет собой газ с резким запахом.
    7. Аммиак действует как …………. агент.
    8. При избытке хлора аммиак реагирует с образованием …………. взрывчатое вещество.
    9. При добавлении к водному раствору медного купороса избытка аммиака …………. образуется цветное соединение.
    10. Чистая азотная кислота становится …………. на стоя.
    11. …………. используется в огнестрельном оружии.
    12. При разложении аммиачной селитры образуется ………….
    13. Белый фосфор бесцветен, но становится бледно-желтым из-за образования …………. стоя.
    14. Белый фосфор можно превратить в …………. нагревом до 420°С в отсутствие воздуха и света.
    15. …………. реагирует со щелочью при кипячении в инертной атмосфере с выделением фосфина.
    16. …………. используется в спичечных коробках.
    17. Фосфин – это …………. запах газа.
    18. Фосфин имеет …………. форма.
    19. …………. используется для создания дымовой завесы.
    20. При гидролизе треххлористого фосфора холодной водой получается ………….
    21. Элементы группы 16 называются ………….
    22. В обычных условиях кислород существует в виде …………. газ.
    23. Аллотрофная форма кислорода …………. а также ………….
    24. Чистый озон – это …………. газ.
    25. …………. используется в сварочных целях.
    26. Моноклинная сера стабильна при температуре от 96° до 119°C и медленно превращается в ………….
    27. …………. газ встречается при извержениях вулканов.
    28. Большое количество …………. газ выбрасывается в атмосферу заводами, использующими уголь и нефть, и медеплавильными заводами.
    29. Газ диоксид серы имеет …………. запах.
    30. Диоксид серы можно использовать для …………. а также …………. в сельском хозяйстве.
    31. В SO 3 S-атом подвергается …………. гибридизация.
    32. В SO 3 двойная связь между S и O возникает из-за …………. перекрытие.
    33. Высокая температура кипения и вязкость серной кислоты обусловлены ………….
    34. …………. используется в качестве осушителя.
    35. Основным источником фтора является ………….
    36. Основным источником хлора является ………….
    37. Хлор – это …………. газ.
    38. Хлор растворим в воде, и его раствор называется ………….
    39. …………. получают пропусканием газообразного хлора через сухую гашеную известь.
    40. …………. используется при добыче золота и платины.
    41. …………. используется для извлечения клея из кости.
    42. При комнатной температуре галогеноводороды являются газами, но …………. можно легко разжижать.
    43. Высвобождение йода, которое дает …………. окрашивание крахмалом.
    44. Каждый галоген соединяется с другими галогенами, образуя ряд соединений, называемых ………….
    45. Структура типа AX 7 ………….
    46. Степень окисления Cl в HClO 4 ………….
    47. Благородные газы имеют …………. энергия ионизации.
    48. Ксенон реагирует с PtF 6 и дает ………….
    49. Kr и газы фтора облучаются SbF, он образует ………….
    50. Форма XeOF 4 ………….
    51. Гелий для наполнения воздуха ………….
    52. …………. используется в люминесцентных лампах.
    53. …………. используется в высокоскоростных электронных лампах-вспышках.
    54. Радон является источником …………. лучи.
    55. …………. образуется при гидролизе мочевины.
    56. …………. элемент subtimes на 889 K.
    57. Желтый фосфор имеет характеристики …………. запах.

    Ответы:

    1. фосфористый
    2. Азот
    3. Инертный
    4. Габера
    5. Азот
    6. Аммиак
    7. Уменьшение
    8. Трихлорид азота
    9. Темно-синий
    10. Желтый
    11. Азотная кислота / NaNO
    12. Закись азота
    13. Слой красного фосфора
    14. Красный фосфор
    15. Белый фосфор
    16. Красный фосфор
    17. Тухлая рыба
    18. Пирамидальный
    19. Фосфин
    20. фосфористая кислота
    21. Халгогены
    22. двухатомный
    23. дикислород и озон
    24. бледно-голубой
    25. Оксиацетилен
    26. Сера ромбическая
    27. Диоксид серы
    28. SO 2
    29. Удушение
    30. дезинфекция сельскохозяйственных культур и растений
    31. сп 2
    32. пπ – дπ
    33. Водородное соединение
    34. Серная кислота
    35. Флюорит
    36. Хлорид натрия
    37. Зелено-желтый
    38. Хлорная вода
    39. Отбеливающий порошок
    40. Хлор
    41. Соляная кислота
    42. фтористый водород
    43. сине-черный
    44. Межгалогенные соединения
    45. пятиугольная бипирамидальная
    46. +7
    47. самый большой
    48. оранжево-желтое твердое вещество [XePtF 6 ]
    49. КрФ 2 . 2СбФ 3
    50. Квадратная пирамида
    51. Воздушные шары
    52. Криптон
    53. Ксенон
    54. Гамма
    55. Аммиак
    56. Мышьяк
    57. Чеснок

    II. Сопоставьте следующее:

    Вопрос 1.
    (i) Процесс Габера – (a) HNO 3
    (ii) Процесс Дикона – (b) Аммиак
    (iii) Контактный процесс – (c) Хлор
    (iv) Процесс Оствальда процесс – (d) H 2 SO 4
    Ответ:
    (i) b
    (ii) c
    (iii) d
    (iv) a

    Вопрос 2.
    (i) Азотная кислота – (a ) Очистка костной сажи
    (ii) HCl – (b) Фотография
    (iii) Белый (желтый) фосфор – (c) Запах тухлой рыбы
    (iv) Фосфин – (d) Фосфоресценция
    Ответ:
    (i) b
    (ii) a
    (iii) d
    (iv) c

    Вопрос 3.
    (i) азотный сесвоксид — (a) H 2 N 2 O 2
    (II) Оксид азота — (B) H 4 N 2 O 4
    (iii) Кислота азотистая – (c) N 2 O
    (iv) Кислота азотистая – (d) N 2 O 3
    Ответ:
    (i) d
    (ii) c
    (iii) a
    (iv) b

    Вопрос 4.
    (i) N 2 O – (a) +5
    (ii) N 2 O 4 – (б) +3
    (iii) N 2 O 5 – (c) +1
    (iv) N 2 O 3 – (d) +4
    Ответ:
    (i ) c
    (ii) d
    (iii) a
    (iv) b

    Вопрос 5.
    (i) Белый фосфор – (a) Извержения вулканов
    (ii) Красный фосфор – (b) Желтый фосфор
    ( iii) Фосфин – (c) Спичечные коробки
    (iv) SO 2 – (d) дымовая завеса
    Ответ:
    (i) b
    (ii) c
    (iii) d
    (iv) a

    Вопрос 6.
    (i) аммиак – (a ) удушливый запах
    (ii) SO 2 – (b) запах тухлой рыбы
    (iii) PH 3 – (c) зеленовато-желтый газ
    (iv) Cl 2 – (d) газ с резким запахом Вопрос 7.0008 2 – (б) зр 3 г 2
    (iii) XeO 3 – (в) зр 3 д 3
    (iv)
    (iv) 2 7 1 9 9 3 XeF 3 9 8 8 8 sp Ответ:
    (i) b
    (ii) d
    (iii) a
    (iv) c

    Вопрос 8.
    (i) Гелий – (a) лампы-вспышки
    (ii) Неон – (b) радиоактивный
    (iii) Крипион – (c) воздушные шары
    (iv) Радон – (d) Ярко-красный
    Ответ:
    (i) c
    (ii) d
    (iii) a
    (iv) b

    IV. Утверждение и причина

    Вопрос 1.
    Утверждение (A) – Ксенон используется в высокоскоростных электронных лампах-вспышках, используемых фотографами.
    Причина (R) – Ксенон мгновенно излучает интенсивный свет в газоразрядных трубках.
    (a) A и R правильные, и R объясняет A
    (b) A и R правильные, но не объясняет A
    (c) A правильный, но R неправильный
    (d) A неправильный, но R правильный
    Ответ:
    (a) A и R верны, и R объясняет A

    Вопрос 2.
    Утверждение (A) – Благородные газы обладают наибольшей энергией ионизации по сравнению с любыми другими элементами.
    Причина (R) – Благородные газы имеют неполную заполненную орбиталь.
    (a) A и R правильные, и R объясняет A
    (b) A и R правильные, но не объясняет A
    (c) A правильный, но R неправильный
    (d) A неправильный, но R правильный
    Ответ:
    (c) A правильно, но R неправильно

    Вопрос 3.
    Утверждение (A) – Иодистый водород разлагается при 4000C, а фтористый и хлористый водород устойчивы при этой температуре. .
    Причина (R) – Термическая стабильность галогеноводородов снижается от фторида к йодиду.
    (a) A и R правильные, и R объясняет A
    (b) A и R правильные, но не объясняет A
    (c) A правильный, но R неправильный
    (d) A неправильный, но R правильный
    Ответ:
    (a) A и R верны и R объясняет A

    Вопрос 4.
    Утверждение (A) – Отбеливание хлором носит временный характер.
    Причина (R) – Хлор окисляет соли двухвалентного железа в соли трехвалентного железа.
    (a) A и R верны, и R объясняет A
    (b) A и R верны, но не объясняет A
    (c) A правильно, но R неправильно
    (d) A неправильно, но R правильно
    Ответ:
    (d) A неправильно, но R правильно

    Вопрос 5.
    Утверждение (A) высоко реактивный.
    Причина (R) – Серная кислота может действовать как сильная кислота и окислитель.
    (a) A и R правильные, и R объясняет A
    (b) A и R правильные, но не объясняет A
    (c) A правильный, но R неправильный
    (d) A неправильный, но R правильный
    Ответ:
    (a) A и R верны, но не объясняют A

    Вопрос 6.
    Утверждение (A) – Серная кислота является высококипящей и вязкой жидкостью.
    Причина (R) — это связано с объединением молекул посредством водородных связей.
    (a) A и R правильные, и R объясняет A
    (b) A и R правильные, но не объясняет A
    (c) A правильный, но R неправильный
    (d) A неправильный, но R правильный
    Ответ:
    (a) A и R верны и R объясняет A

    Вопрос 7.
    Утверждение (A) – Моноклинная сера менее стабильна, чем ромбическая сера.
    Причина (R) – Моноклинная сера стабильна при температуре от 96°C до 119°C и медленно превращается в ромбическую серу.
    (a) A и R правильные, и R объясняет A
    (b) A и R правильные, но не объясняет A
    (c) A правильный, но R неправильный
    (d) A неправильный, но R правильный
    Ответ:
    (a) A и R верны, и R объясняет A

    Вопрос 8.
    Утверждение (A) – Газообразный азот химически инертен.
    Причина (R) – Азот имеет низкую энергию связи.
    (a) A и R верны, и R объясняет A
    (b) A и R верны, но не объясняют A
    (c) A верно, но R неверно
    (d) A неверно, но R верно
    Ответ:
    (c) A верно, но R неверно неправильно

    V. Найдите лишнее
    Вопрос 1.
    (a) NO
    (b) HNO 3
    (C) NO 2
    (d) N 2 O
    Ответ:
    ) HNO 3
    Подсказка: HNO 3 – это кислота, а другие – оксиды.

    Вопрос 2.
    (a) Азотистая кислота
    (b) Азотная кислота
    (c) Азотистоватая кислота
    (d) Пемитронистая кислота
    Ответ:
    (d) Петриевая кислота
    Подсказка: Пемитронистая кислота содержит перекисную связь, другие не имеют перекисной связи.

    Вопрос 3.
    (а) Белый фосфор
    (б) Красный фосфор
    (в) пентаоксид фосфора
    (г) черный фосфор
    Ответ:
    (в) пентаоксид фосфора
    Подсказка: P 2

    9 O 9

    представляет собой соединение фосфора, а другие представляют собой аллотропную форму фосфора.

    Вопрос 4.
    (a) PH 3
    (b) HPO 3
    (C) H 3 PO 3
    (D) H 3 PO 4
    Ответ:
    (H a) PH 3
    Подсказка: PH 3 представляет собой гидриды фосфора, а другие представляют собой оксокислоты фосфора.

    Вопрос 5.
    (a) He
    (b) Ne
    (c) Ar
    (d) Xe
    Ответ:
    (d) Xe
    Подсказка: Xe образует несколько химических соединений, чем другие.

    VI. Найдите правильную пару.

    Вопрос 1.
    (a) Гелий – лампы накаливания
    (b) Криптон – предотвращающие соединения
    (c) Ксенон – лазеры
    (d) Радон – лампы-вспышки
    Ответ:
    (c) Ксенон – лазеры

    Вопрос 2
    (a) Ra – гамма-лучи
    (b) Xe – раковые образования
    (c) Ne – воздушные шары
    (d) Kr – рекламная лампочка
    Ответ:
    (a) Ra – гамма-лучи

    Вопрос 3.
    (a) ClF 3 – Линейная
    (b) BrF 5 – Т-образная
    (c) IF 4 – квадратная пирамидальная
    (г) BrF 5 – квадратная пирамидальная
    Ответ:
    (г) BrF 5 – квадратная пирамидальная

    Вопрос 4. ) XEF 6 — SP 3 D 3
    (C) XEF 4 — SP 3
    (D) XEOF 4 — SP 3 D
    ОТВЕТ:
    (B) B). 6 – sp 3 d 3

    Вопрос 5.
    (a) OF 2 = -1
    (б) Кл 4 О 4 = -1
    (в) I 2 О 4 = -1
    (г) I 2 О

    8 9 9 Ответ:


    (a) OF 2 = -1

    Вопрос 6.
    (a) Королевская вода – Растворение золота
    (b) Хлор – Выделение клея из кости
    (c) HCl – Извлечение золота
    (d ) Хлор – Временное отбеливание
    Ответ:
    (а) Королевская вода – Растворение золота

    Вопрос 7.
    (а) О – 2с 2 2p 3
    (b) S – 3s 2 3p 4
    (c) Se – 4d 10 5s 2 5p 4
    (d) Te – 3d 10 4s 2 4p 4
    Ответ:
    (b) S – 3s 2 3p 4

    VI. Найдите неправильную пару.

    Вопрос 1.
    (a) Газообразный азот – инертный
    (b) Аммиак – газ с резким запахом
    (c) Азотная кислота – окислитель
    (d) Фосфин – запах тухлых яиц
    Ответ:
    (d) Фосфин – запах тухлых яиц

    Вопрос 2.
    (a) Жидкий азот – биологическая консервация
    (b) Азотная кислота – фотография
    (c) белый фосфор – желтый фосфор
    (d) фосфор – сварка
    Ответ:
    (г) фосфор – сварка

    Вопрос 3.
    (а) N 2 O = +1
    (b) N 2 O = +2
    (c) N

    8 2

    8 2

    8

    3 = +5
    (г) НЕТ 2 = +4
    Ответ:
    (в) N 2 O 3 = +5

    Вопрос 4.
    а) азотистая кислота – N 2 O
    б) азотистая кислота – HNO 2
    в) перазотная кислота – HNO 9 0 9 9 0 9 0 9 0 9 0 4 (г) перазотистая кислота – HOONO
    Ответ:
    (а) Нвпонитистая кислота – N 2 O

    Вопрос 5. c) H 2 SO 4 – Дезинфекция посевов
    (d) SO 3 – Осветление волос
    Ответ:
    (c) H 2 SO 4 – Дезинфекция сельскохозяйственных культур

    Вопрос 6.
    (a) H 2 SO 4 осушитель
    (b) HCl 7c – Процесс Дикона – очистка костяной сажи
    (d) Гелий – лампы-вспышки
    Ответ:
    (d) Гелий – лампы-вспышки

    Вопрос 7.
    (a) ICl – линейная
    (b) CIF 3 – Т-образная
    (c) IF 5 – пентагонально-бипирамидальный
    (d) IF 7 – пентагонально-бипирамидальный
    Ответ:
    (в) IF 5 – пятиугольная бипирамида

    Вопрос 8.
    (а) HOCl = +2
    (б) HOCl = +3
    (д) HOCI = +5
    (г) HOCl 4 = +7
    Ответ:
    (а) HOCl = +2

    Вопрос 9.
    (а) XeF 4 – sp 3 d 3
    (б) XeF 9 93 9090 6 d 3
    (e) XeOF 4 – sp 3 d 2
    (d) XeO 3 – sp 3 d 7 Ответ:2117 (d) XeO 3 – sp 3 d

    Вопрос 10.
    (a) He – криогеника
    (b) Ne – реклама
    (c) Kr – люминесцентные лампы
    (d) Ra – лазеры.
    Ответ:
    (г) Ра – Лазеры.

    Samacheer Kalvi 12th Chemistry p-Block Elements – II 2 балла Вопросы и ответы

    Вопрос 1.
    Что такое пниктогены?
    Ответ:
    Группа из 15 элементов, таких как азот, фосфор, мышьяк, сурьма и висмут, в совокупности называются пниктогенами. Их общая внешняя электронная конфигурация ns 2 нп 3 .

    Вопрос 2.
    Что происходит при термическом разложении азида натрия?
    Ответ:
    Чистый газообразный азот можно получить термическим разложением азида натрия около 575K
    2NaN 3 \(\underrightarrow { 575K }\) 2Na + 3N 2

    Вопрос 3.
    Как приготовить аммиак из азота? и указать название процесса?
    Ответ:
    Азот непосредственно реагирует с водородом с образованием аммиака. Этой реакции благоприятствуют высокие давления и оптимальная температура в присутствии железного катализатора,
    N 2 + 3H 3 \(\rightleftharpoons\) 2NH 3
    Этот процесс называется процессом Габера.

    Вопрос 4.
    Почему газообразный азот химически инертен?
    Ответ:
    Химически инертный характер азота в значительной степени обусловлен высокой энергией связи молекул 225 кал моль -1 . Интересно, что частицы с тройной связью отличаются меньшей реакционной способностью по сравнению с другими изоэлектронными системами с тройной связью, такими как как -C \(\equiv\) C – C \(\equiv\) Q, X- C \(\equiv\) N и т. д.

    Вопрос 5.
    Укажите использование азота?
    Ответ:

    1. Азот используется для производства аммиака, азотной кислоты, цианамида кальция и т. д.
    2. Жидкий азот используется для создания низкой температуры, необходимой в криохирургии, а также для биологической консервации

    Вопрос 6.
    Объясните действие тепла на аммиак.
    Ответ:
    Выше 500°C аммиак разлагается на элементы. Разложение может быть ускорено металлическими катализаторами, такими как никель, железо. Практически полная диссоциация происходит при непрерывном искрении. 9{ 0 }C } \) N 2 + 3H 2

    Вопрос 7.
    Докажите, что аммиак действует как восстановитель?
    Ответ:
    Аммиак восстанавливает оксиды металлов до металла при пропускании через нагретый оксид металла.
    3PbO + 2NH 3 → 3Pb + N 2 + 3H 2 O

    Вопрос 8.
    Что происходит при реакции сульфата меди с аммиаком?
    Ответ:
    При добавлении избытка аммиака к водному раствору сульфата меди появляется темно-синий цвет Комплекс [Cu(NH 3 ) 4 ] 2+ .

    Вопрос 9.
    Чистая азотная кислота бесцветна, при стоянии становится желтой. Обосновать ответ.
    Ответ:
    Азотная кислота под действием солнечного света или при нагревании разлагается на двуокись азота, воду и кислород.
    4HNO 4 → 4NO 2 + 2H 2 O + O 2
    Благодаря этой реакции чистая кислота или ее концентрированный раствор при стоянии желтеют.

    Вопрос 10.
    Напишите продукты, образующиеся при взаимодействии азотной кислоты с разбавленным и концентрированным с магнием.
    Ответ:
    1. Магний с конусом. HNO 3 :
    4 мг + 10HNO 3 → 4 мг (№ 3 ) 2 + NH 4 NO 3 + 3H 2 o
    Реакты магния с концентрированной аид -кислотой для оба магнела NITRAT и нитрат аммония.

    2. Магний с разбавленной HNO 3 :
    Магний реагирует с разбавленной азотной кислотой с образованием нитрата магния и закиси азота.
    4Mg + 10HNO 3 → 4 Mg(NO 3 ) 2 + N 2 O + 5H 2 O

    Вопрос 11.
    Укажите применение азотной кислоты.
    Ответ:

    1. Азотную кислоту применяют как окислитель и при приготовлении акварегионов.
    2. 4. Соли азотной кислоты используются в фотографии (AgNO 3 ) и в качестве пороха для огнестрельного оружия. (NaNO 3 ).

    Вопрос 12.
    Как вы будете получать оксид азота из нитрита натрия?
    Ответ:
    При взаимодействии нитрита натрия с сульфатом железа в присутствии серной кислоты образуется окись азота.
    2NANO 2 + 2FESO 4 + 3H 2 S → FE 2 (SO 4 ) 3 + 2NAHSO 4 + 2H 2 O + 2NO

    9003

    .
    Как приготовить пятиокись азота?
    Ответ:
    Азотная кислота реагирует с пятиокисью фосфора с образованием пятиокиси азота.
    2ХНО 3 + P 2 O 5 → N 2 O 5 + 2HPO 3

    Вопрос 14.
    Нарисуйте структуру

    1. N 2 O
    2. Н 2 О 3

    Ответ:
    1. N 2 O (оксид азота)

    2. N 2 O 3 (Sesquoxide азота)

    Вопрос 15.
    Dall- Структура

    4
  • N 888888888.
    . 2
    О 4
  • Н 2 О 3
  • Ответ:
    1. N 2 O 4 (азот тетраоксид)

    2. N 2 O 5

    Вопрос 16.
    . Нарисуйте структуру

    .

  • Азотоводородная кислота.
  • Ответ:
    (a) Гипоноватистая кислота – H 2 N 2 O 2
    HO – N = H – OH

    (b) Азотистая кислота:

    Вопрос 17.
    Назовите аллотропные формы фосфора?
    Ответ:
    Наиболее распространенные аллотропные формы фосфора

    1. белый фосфор
    2. Красный фосфор
    3. Черный фосфор

    Вопрос 18.
    Почему белый фосфор также известен как желтый фосфор?
    Ответ:
    Свежеприготовленный белый фосфор бесцветен, но при стоянии становится бледно-желтым из-за образования слоя красного фосфора. Поэтому он также известен как желтый фосфор.

    Вопрос 19.
    Что такое фосфоресценция?
    Ответ:
    Белый (желтый) фосфор светится в темноте за счет окисления, называемого фосфоресценцией.

    Вопрос 20.
    Почему белый фосфор самовозгорается на воздухе?
    Ответ:
    Температура воспламенения белого фосфора очень низкая, поэтому он самовозгорается на воздухе при комнатной температуре, а в процессе горения белый фосфор образует P 2 O 5 .

    Вопрос 21.
    Нарисуйте структуру

    1. белый фосфор
    2. красный фосфор

    Ответ:
    1. Белый фосфор

    2. Красный фосфор

    Вопрос 22.
    Как вы преобразуете красный фосфор в P 2 O 3 и P 2 Q 5 ?
    Ответ:
    Красный фосфор реагирует с кислородом при нагревании с образованием трехокиси или пятиокиси фосфора.
    P 4 + 3O 2 \(\underrightarrow { \triangle }\) P 4 O 6 (триоксид фосфора)
    P 4 + 5O 2 (\underright \triangle } \) P 4 Q 10 (пентаоксид фосфора)

    Вопрос 23.
    Как вы будете получать ортофосфорную кислоту из фосфора?
    Ответ:
    При обработке фосфора конц. азотной кислоты окисляется до ортофосфорной кислоты. Эта реакция катализируется кристаллами йода.

    Вопрос 24.
    Укажите использование фосфора?
    Ответ:

    1. Красный фосфор используется в спичечных коробках.
    2. Он также используется для производства некоторых сплавов, таких как фосфористая бронза.

    Вопрос 25.
    Показать, что фосфин является слабоосновным?
    Ответ:
    Фосфин слабоосновен и образует соли фосфония с галогенокислотами.
    PH 3 +HI → PH 4 I
    PH 4 I + H 2 O \(\underrightarrow { \triangle }\) PH 3 + H 3 O + I

    Вопрос 26.
    Ответ: Изобразите структуру PCl 9 217 9 9217 3

    Вопрос 27.
    Как вы будете получать PCl 3 из белого фосфора?

    1. При пропускании медленного потока хлора над белым фосфором образуется PCl 3
    2. Его также можно получить обработкой белого фосфора тионилхлоридом.

    Вопрос 28.
    Что произойдет, если PCl 3 обработать холодной водой?
    Ответ:
    При гидролизе PCl 3 холодной водой получается фосфористая кислота.

    Вопрос 29.
    Как вы будете готовить PCl 3 ?
    Ответ:
    При обработке PCl 3 избытком хлора получается пентахлорид фосфора.
    PCl 3 + Cl 2 → PCl 5

    Вопрос 30.
    Укажите использование PCl 3 и PCl 5 ?
    ответ:

    1. Трихлорид фосфора используется в качестве хлорирующего агента и для получения H 2 PO 3 .
    2. Пентахлорид фосфора является хлорирующим агентом и полезен для замены гидроксильных групп атомами хлора.

    Вопрос 31.
    Нарисуйте структуру H 3 PO 2 и H 3 PO 3 ?
    Ответ:

    Вопрос 32.
    Нарисуйте структуру H 4 P 2 O 6 и H 4 P 2 O 3 ?
    ANSSWER:
    (I) H 4 P 2 O 6
    (II) H 4 P 2 O 3

    Вопрос 33.
    Дайте метод для подготовки гипофосфорозной кислоты. а пирофосфорная кислота?
    Ответ:
    1. Фосфорноватистая кислота – Фосфор реагирует с водой с образованием фосфорноватистой кислоты.

    2. Пирофосфорная кислота – При нагревании фосфорной кислоты образуется пирофосфорная кислота.

    Вопрос 34.
    Завершите реакции

    1. HgO \(\underrightarrow { \triangle }\) ?
    2. BaO 2 \(\underrightarrow {\треугольник}\) ?

    Ответ:

    1. HgO \(\underrightarrow { \triangle }\) 2Hg + O 2
    2. 2BaO 2 \(\underrightarrow { \triangle }\) 2BaO + O 2

    Вопрос 35.
    Приведите и объясните реакцию, используемую для определения озона.
    Ответ:
    Озон окисляет йодид калия до йода.
    O 3 + 2KI + H 2 O → 2KOH + O 2 + I 2
    Эта реакция является количественной и может использоваться для оценки озона.

    Вопрос 36.
    Напишите краткие заметки о ромбической сере?
    Ответ:
    Ромбическая сера, также известная как сера, является единственной термодинамически стабильной аллотропной формой при обычных температуре и давлении. Кристаллы имеют характерный желтый цвет и состоят из S 8 молекул. При медленном нагревании выше 96°С превращается в моноклинную серу. При охлаждении ниже 96°C β-форма снова превращается в форму.

    Вопрос 37.
    Напишите заметку о моноклинной сере?
    Ответ:
    Моноклинная сера также содержит молекулы S 8 в дополнение к небольшому количеству молекул S 6 . Она существует в виде длинной игольчатой ​​призмы и также называется призматической серой. Он стабилен при температуре от 96° до 119°C и медленно превращается в ромбическую серу.

    Вопрос 38.
    Что такое λ – сера?
    Ответ:
    Сера также существует в жидком и газообразном состояниях. При температуре около 140° C моноклинная сера плавится с образованием подвижной бледно-желтой жидкости, называемой серой λ.

    Вопрос 39.
    Как вы будете получать диоксид серы лабораторным методом?
    Ответ:
    Двуокись серы получают в лаборатории, обрабатывая металл или сульфит металла серной кислотой.
    Cu + 2H 2 SO 4 → CuSO 9{ – }\) + 2H + → H 2 O + SO 2

    Вопрос 40.
    Закончить реакции,
    Ответ:

    1. Zns + O 2 }\) ?
    2. FeS 2 + O 2 \(\underrightarrow {\треугольник}\) ?

    Ответ:

    1. 2Zns + 3O 2 \(\underrightarrow { \triangle }\) 2ZnO + 2SO 2
    2. 4FeS 2 + 11O 2 \(\underrightarrow { \triangle }\) 2Fe 2 О 3 + 8СО 2

    Вопрос 41.
    Что происходит, когда диоксид серы реагирует с гидроксидом натрия и карбонатом натрия?
    Ответ:
    Диоксид серы реагирует с гидроксидом натрия и карбонатом натрия с образованием бисульфита натрия и сульфита натрия соответственно.

    Вопрос 42.
    Укажите использование диоксида серы?
    Ответ:

    1. Двуокись серы используется для обесцвечивания волос, шелка, шерсти и т. д.
    2. Может использоваться для дезинфекции сельскохозяйственных культур и растений.

    Вопрос 43.
    Почему серная кислота является высококипящей и вязкой жидкостью?
    Ответ:
    Серная кислота является высококипящей и вязкой жидкостью, это происходит из-за объединения молекул посредством водородных связей.

    Вопрос 44.
    Объясните реакцию между бензолом и серной кислотой.
    Ответ:
    Серная кислота реагирует с бензолом с образованием бензолсерной кислоты.

    Вопрос 45.
    Назовите применение серной кислоты?
    Ответ:

    1. Серная кислота используется в производстве удобрений, сульфата аммония и суперфосфатов, а также других химических веществ, таких как соляная кислота, азотная кислота и т. д.
    2. Используется в качестве осушителя, а также при приготовлении пигментов, взрывчатых веществ и т. д.

    Вопрос 46.
    Изобразите структуру H 2 SO 3 и H 2 SO 4 ?
    Ответ:
    1. H 2 SO 3

    2. H 2 SO 4

    Вопрос 48.
    Нарисуйте структуру

    1. Pyrosulphuric Artic
    2. 4444444444444444444.
    3. Пероромоносерная кислота?
    Samacheer Kalvi 12th Chemistry Solutions Chapter 3 p-Block Elements – II

    Ответ:
    1. Пиросерная кислота H 2 S 2 O 7

    2. Пероромоносерная кислота

    009 SO 5

    Вопрос 49.
    Что происходит при взаимодействии хлора со скипидаром?
    Ответ:
    При сгорании хлора со скипидаром образуется углерод и соляная кислота.

    Вопрос 50.
    Как вы будете получать соляную кислоту лабораторным методом?
    Ответ:
    Получается действием хлорида натрия и концентрированной серной кислоты

    • NaCl + H 2 SO 4 → NaHSO 4 + HCl
    • NaHSO 4 + NaCl → Na 2 SO 4 + HCl

    Кислота соляная сухая получается пропусканием газа через конц. серная кислота

    Вопрос 51.
    Укажите применение соляной кислоты?
    Ответ:

    1. Соляная кислота используется для производства хлора, хлорида аммония, глюкозы из кукурузного крахмала и т.д.,
    2. Используется для извлечения клея из кости, а также для очистки костной сажи.

    Вопрос 52.
    Проведите следующую реакцию?
    Ответ:

    Вопрос 53.
    Почему инертные газы обладают наибольшей энергией ионизации?
    Ответ:
    Благородные газы обладают наибольшей энергией ионизации по сравнению с любыми другими элементами в данном ряду, поскольку они полностью заполняют орбитали (ns 2 np 6 ) в самой внешней оболочке. Они чрезвычайно стабильны и имеют небольшую тенденцию приобретать или терять электроны. Следовательно, благородные газы обладают наибольшей энергией ионизации.

    Вопрос 54.
    Расскажите об использовании неона?
    Ответ:
    Неон используется в рекламе как неоновая вывеска, а яркое красное свечение возникает при пропускании электрического тока через газ неон под низким давлением
    .

    Вопрос 55.
    Назовите способы использования Криптона?
    Ответ:

    1. Криптон используется в люминесцентных лампах, лампах-вспышках и т. д.
    2. Лампы с криптоном используются в аэропортах в качестве ближних огней, так как они могут проникать сквозь густой туман.

    Вопрос 56.
    Упомяните применение ксенона?
    Ответ:

    1. Ксенон используется в люминесцентных лампах, импульсных лампах и лазерах.
    2. Ксенон мгновенно излучает интенсивный свет в газоразрядных трубках. В связи с этим он используется в высокоскоростных электронных вспышках, используемых фотографами.

    Вопрос 57.
    Назовите применение радона?
    Ответ:

    1. Радон радиоактивен и используется как источник гамма-излучения
    2. Газообразный радон
    3. запечатывается в виде небольших капсул и имплантируется в организм для уничтожения злокачественных, т.е. раковых, новообразований.

    Вопрос 58.
    Почему пентагалогениды более ковалентны, чем тригалогениды?
    Ответ:
    Поскольку элементы в степени окисления +5 менее склонны терять электроны, чем в степени окисления +3, поэтому элементы в степени окисления +5 более ковалентны, чем в степени окисления +3. Другими словами, пентагалогениды более ковалентны, чем тригалогениды.

    59. Почему N 2 менее реактивен при комнатной температуре?
    Ответ:
    Из-за наличия тройной связи между двумя атомами азота энергия диссоциации связи N 2 (941,4 кДж моль ) очень высока. Следовательно, N 2 менее реакционноспособен при комнатной температуре.

    Вопрос 60.
    Почему ICl более реактивен, чем I 2 ?
    Ответ:
    Связь ICl слабее связи I–I. Поэтому ICl может легко разрушаться с образованием атомов галогена, которые легко вызывают реакцию.

    Вопрос 61.
    Почему в водолазных аппаратах используется гелий?
    Ответ:
    Он используется в качестве разбавителя кислорода в современных водолазных аппаратах из-за его очень низкой растворимости в крови.

    Вопрос 62.
    Приведите реакцию диспропорционирования H 3 PO 3 ?
    Ответ:
    H 3 PO 3 при нагревании подвергается самоокислению:

    Вопрос 63.
    Почему красный фосфор менее активен, чем белый фосфор?
    Ответ:
    Белый фосфор более реакционноспособен, чем красный фосфор при нормальных условиях из-за угловой деформации в молекуле P 4 , где углы составляют всего 60°.

    Вопрос 64.
    Азот не образует никаких пентагалогенидов, как фосфор. Почему?
    Ответ:
    Азот не образует пентагалогенидов из-за отсутствия d-орбиталей в его валентной оболочке.

    Вопрос 65.
    Обоснуйте следующее. Из инертных газов хорошо известно, что только ксенон образует химические соединения?
    Ответ:
    Xe имеет самый большой размер и самую высокую поляризующую способность.

    Вопрос 66.
    Почему сероводород с большей молекулярной массой является газом, а вода – жидкостью при комнатной температуре?
    Ответ:
    H 2 Молекулы O связаны межмолекулярными Н-связями, H 2 S — нет, так как кислород более электроотрицателен и меньше по размерам, чем сера. Вот почему H 2 O — жидкость, а H 2 S — газ.

    Вопрос 67.
    Благородные газы химически инертны. Назови причины.
    Ответ:
    Благородные газы химически инертны, потому что они имеют полный октет, кроме гелия, т.е. они имеют стабильную электронную конфигурацию.

    Вопрос 68.
    Почему инертные газы существуют как одноатомные?
    Ответ:
    Благородные газы имеют стабильную электронную конфигурацию, поэтому они не склонны терять или приобретать электроны. Поэтому они не образуют ковалентной связи.

    Вопрос 69.
    Азот существует в виде двухатомной молекулы, а фосфор – в виде P 4 . Почему?
    Ответ:
    Азот имеет тройную связь между двумя атомами из-за его небольшого размера и высокой электроотрицательности. Фосфор P 4 имеет одинарную связь, поэтому он четырехатомный.

    Вопрос 70.
    Почему H 2 S более кислая, чем вода?
    Ответ:
    Это связано с тем, что энергия диссоциации связи S-H меньше, чем связь O-H из-за большей длины связи.

    Samacheer Kalvi 12th Chemistry p-Block Elements – II 3 балла Вопросы и ответы

    Вопрос 1.
    Закончите следующие реакции.
    (i) 6Li + N 2 → ?
    (ii) 3Ca + N 2 { +} \) + OH → 2NH 3 + H 2 O
    2NH 4 CL + CAO → CACL 2 + 2NH 3 + H 2 O

    3. ITH 3 + H 2 O

    . также можно получить путем нагревания нитридов металлов, таких как нитрид магния, с водой.
    Mg 3 N 2 + 6H 2 O → 3Mg(OH) 2 + 2NH 3

    Вопрос 3.
    Объясните строение аммиака.
    Ответ:
    Молекула аммиака имеет пирамидальную форму, расстояние связи N-H равно 1,016 A, расстояние связи H-H равно 1,645 A с валентным углом 107°. Структуру аммиака можно рассматривать как тетраэдрическую с одной неподеленной парой электронов в одном тетраэдрическом положении, поэтому она имеет пирамидальную форму, как показано на рисунке.

    Вопрос 4.
    Как вы будете получать азотную кислоту?
    Ответ:
    Азотную кислоту получают нагреванием равных количеств нитрата калия или натрия с концентрированной серной кислотой.
    KNO 3 + H 2 SO 4 → KHSO 4 + HNO 3

    Температуру поддерживают как можно более низкой, чтобы избежать разложения азотной кислоты. Кислота конденсируется в дымящуюся жидкость, окрашенную в коричневый цвет из-за присутствия небольшого количества диоксида азота, образующегося при разложении азотной кислоты.
    4HNO 3 → 4NO 2 + 2H 2 O + O 2

    Вопрос 5.
    Обсудите коммерческий способ получения азотной кислоты.
    (или)
    Как вы будете получать азотную кислоту по способу Оствальда?
    Ответ:
    Азотная кислота, полученная в больших масштабах с использованием процесса Оствальда. В этом методе аммиак из процесса Габера смешивается примерно в 10 раз с воздухом. Эта смесь предварительно нагревается и поступает в каталитическую камеру, где контактирует с платиновой сеткой. Температура повышается примерно до 1275 К, и металлическая сетка вызывает быстрое каталитическое окисление аммиака, приводящее к образованию NO, который затем окисляется до двуокиси азота.
    4NH 3 + 5O 2 → 4NO + 6H 3 O + 120 кДж
    2NO + O 2 → 2NO 3

    производится через колонну адсорбции диоксидов азота. Реагирует с водой с образованием азотной кислоты. Образовавшуюся азотную кислоту отбеливают продувкой воздухом.
    6NO 2 + 3H 2 O → 4HNO 3 + 2NO + H 2 O

    Вопрос 6.
    Изобразите строение следующих соединений.
    (а) Азотистая кислота
    (b) Азотная кислота
    (c) Перазотная кислота
    Ответ:

    Вопрос 7.
    Определите A, B и C из следующих реакций.

    1. P 4 + Mg → A
    2. P 4 + Ca → B
    3. P 4 + Na → C

    Ответ:

    1. P 4 + 6Mg → 2Mg 3 P 2 (фосфид магния)
    2. P 4 + 6Ca → 2Ca 3 P 2 (фосфид кальция)
    3. P 4 + 12Na → 2Na 3 P (фосфид натрия)

    Вопрос 8.
    Как вы приготовите фосфин и объясните процесс очистки фосфина?
    Ответ:
    Фосфин получают действием гидроксида натрия с белым фосфором в инертной атмосфере углекислого газа или водорода.

    Фосфин выделяют из дигидрида фосфина (P 2 H 4 ) пропусканием через замораживающую смесь. Дигидрид конденсируется, а фосфин — нет. Фосфин также можно получить гидролизом фосфидов металлов водой или разбавленными минеральными кислотами.

    Фосфин получают в чистом виде нагреванием фосфористой кислоты.

    Образец чистого фосфина получают путем нагревания йодистого фосфония с раствором едкого натра.

    Вопрос 9.
    Что происходит, когда PH 3 реагирует с кислородом или воздухом?
    Ответ:
    При попадании фосфина в воздух или кислород он сгорает с образованием метафосфорной кислоты.

    Вопрос 10.
    Объясните строение фосфина.
    Ответ:
    В фосфине фосфор показывает sp 3 гибридизация. Три орбитали заняты парой связей, а четвертый угол занят неподеленной парой электронов. Следовательно, валентный угол уменьшается до 94°. Фосфин имеет пирамидальную форму.

    Вопрос 11.
    Обсудите применение фосфина.
    Ответ:
    Фосфин используется для создания дымовой завесы, так как дает большой дым. На корабле проткнутый контейнер со смесью карбида кальция и фосфида кальция высвобождает фосфин и ацетилен при выбросе в море. Высвобожденный фосфин воспламеняется и воспламеняет ацетилен. Эти горящие газы служат сигналом для приближающихся кораблей. Это известно как сигнал Холмса.

    Вопрос 12.
    Как PCl 3 реагирует со следующими реагентами?

    1. С 2 Н 5 ОХ
    2. С 2 Н 5 СООН

    Ответ:

    1. C 2 H 5 OH + PCL 3 → C 2 H 5 CL + H 3 PO 3
    2. C 2 H 5 COOH + PCl 3 → C 2 H 5 COCl + H 3 Заказ на поставку 3

    Вопрос 13.
    Объясните реакцию между PCl 5 и водой.
    Ответ:
    Пентахлорид фосфора реагирует с водой с образованием фосфорилхлорида и ортофосфорной кислоты.

    Вопрос 14.
    Объясните строение трехокиси фосфора (P 2 O 3 ).
    Ответ:
    В трехокиси фосфора четыре атома фосфора лежат по углам тетраэдра и шесть атомов кислорода по краям. Расстояние связи P – O составляет 165,6 пм, что меньше, чем расстояние одинарной связи P – O (184 пм) из-за связи pπ – dπ и приводит к значительному характеру двойной связи.

    Вопрос 15.
    Обсудите структуру пентаоксида фосфора (P 2 O 3 ).
    Ответ:
    В Р 4 О 10 каждый атом Р образует три связи с атомом кислорода, а также дополнительную координационную связь с атомом кислорода. Конечная координата длины связи P – O составляет 143 пм, что меньше ожидаемого расстояния одинарной связи. Это может быть связано с боковым перекрытием заполненных р-орбиталей атома кислорода с незанятой d-орбиталью фосфора.

    Вопрос 16.
    Укажите использование кислорода.
    Ответ:

    1. Кислород является одним из основных компонентов для выживания живых организмов.
    2. Используется при сварке (ацетиленовой сварке)
    3. Жидкий кислород используется в качестве топлива в ракетах и ​​т. д.

    Вопрос 17.
    Дайте и объясните восстановительные свойства двуокиси серы.
    Ответ:
    Поскольку двуокись серы легко окисляется, она действует как восстановитель. Восстанавливает хлор в соляную кислоту.
    SO 2 + 2H 2 O + Cl 2 → H 2 SO 4 + 2HCl
    Также восстанавливает перманганат и дихромат калия до Mn

    4 2+ соответственно

    • 2KMNO 4 + 5SO 2 + 2H 2 O → K 2 SO 4 + 2MNSO 4 + 2H 2 SO 4
    • 4 40009 + 2H 2 SO 4 4 + 2H 2
    • K 2 Cr 2 O 7 + 3SO 2 + H 2 SO 4 → K 2 SO 4 + Cr 2 (SO 4 ) 3 + H 2 O

    Вопрос 18.
    Почему отбеливающее действие диоксида серы временное?
    Ответ:
    В присутствии воды диоксид серы обесцвечивает окрашенную шерсть, шелк, губки и солому из-за своих восстановительных свойств.
    SO 2 + 2H 2 O → 2H 2 SO 4 + 2(H)

    Однако отбеленный продукт (бесцветный) выдерживают на воздухе, он повторно окисляется кислородом воздуха до свой первоначальный цвет. Следовательно, отбеливающее действие диоксида серы носит временный характер.

    Вопрос 19.
    Объясните строение диоксида серы.
    Ответ:
    В диоксиде серы атом серы подвергается sp 2 гибридизации. Двойная связь между S и O возникает за счет pπ-dπ-перекрывания.

    Вопрос 20.
    Как вы будете производить серную кислоту контактным способом?
    Ответ:
    Производство серной кислоты контактным способом включает следующие этапы:
    1. Первоначально двуокись серы получают путем сжигания сернистого или железного колчедана в кислороде/воздухе.
    S + O 2 → SO 2
    4FES 2 + 11O 2 → 2FE 2 O 3 + 8SO 2

    2. Формируется диоксид серы. в присутствии катализатора, такого как V 2 O 5 или платинированного асбеста.

    3. Триоксид серы абсорбируется концентрированной серной кислотой и дает олеум (H 2 S 2 O 7 ). Олеум превращают в серную кислоту, разбавляя его водой.

    Для достижения максимальной производительности установка работает при давлении 2 бар и температуре 720 K. Серная кислота, полученная в этом процессе, имеет чистоту более 96 %.

    Вопрос 21.
    Докажите, что H 2 SO 4 — сильная двухосновная кислота.
    Ответ:
    Серная кислота образует два типа солей, а именно сульфаты и бисульфаты.

    Ответ:

    Вопрос 23.
    Что происходит при взаимодействии хлора с аммиаком?
    Ответ:
    1. Если хлор реагирует с избытком аммиака, то получается азот и хлористый аммоний.
    8NH 3 + 3Cl 2 → N 2 + 6NH 4 Cl

    2. Если аммиак реагирует с избытком хлора, он дает трихлорид азота и хлорид аммония.
    4NH 2 + 3Cl 2 → NCl 2 + 3NH 4 Cl

    Вопрос 24.
    Отбеливающее действие хлора является постоянным. Обоснуйте это утверждение.
    Ответ:
    Хлор является сильным окислителем и отбеливателем из-за образующегося кислорода.

    Красящее вещество + выделяющийся кислород → Бесцветный продукт окисления
    Следовательно, отбеливание хлором является необратимым. Он окисляет соли двухвалентного железа до трехвалентного, сульфиты до сульфатов и сероводород до серы.

    Вопрос 25.
    Укажите применение хлора.
    Ответ:

    9{ 2- }\) + NO + 8H 2 O

    Вопрос 27.
    Почему HF не хранят в стеклянных бутылях?
    Ответ:
    HF быстро реагирует с кремнеземом и стеклом с образованием растворимой соли, и этот процесс приводит к разбиванию стеклянных бутылок. Поэтому HF не хранят в стеклянных бутылках.
    SIO 2 + 4HF → SIF 4 + 2H 2 O
    NA 2 SIO 3 + 6HF → NA 2 6 + 3H 2 O

    9003 + 3H 2 O

    + 3H 2 O

    3 + 3H 2 O

    3 + 3H 2 O 3 + 3H 2 O 3 + 3H 2 O 8 + 3H 2 .
    Назовите свойства межгалогеновых соединений.
    Ответ:
    Свойства межгалогеновых соединений:

    1. Центральный атом будет больше.
    2. Может образовываться только между двумя галогенами и не более чем двумя галогенами.
    3. Фтор не может выступать в качестве центрального атома металла, будучи самым маленьким.
    4. Из-за высокой электроотрицательности при небольшом размере фтор помогает центральному атому достичь высокого координационного числа
    5. Могут подвергаться автоионизации.
    6. Являются сильными окислителями.

    Вопрос 29.
    Как приготовить фторид ксенона?
    Ответ:
    Фториды ксенона получают прямой реакцией ксенона и фтора в различных условиях, как показано ниже.

    Вопрос 30.
    Закончите следующую реакцию.
    (i) XEOF 4 + SIO 2 → A + SIF 6
    (ii) A + SIO 2 → B + SIF 6
    Ответ:

    Вопрос 31.
    Что происходит, когда происходит, когда происходит, когда происходит, когда происходит, когда происходит, когда происходит, когда происходит, когда происходит, когда происходит, когда происходит, когда происходит, когда происходит XEF 6 реагирует с 2,5 М раствором NaOH?
    Ответ:
    При взаимодействии XeF 6 с 2,5 М NaOH образуется перксенат натрия.

    Вопрос 32.
    Приведите два примера, показывающих аномальное поведение фтора.
    Ответ:
    Аномальное поведение фтора связано с его:

    1. Малым размером
    2. наивысшая электроотрицательность
    3. низкая энтальпия диссоциации связи F-F
    4. отсутствие d-орбиталей в его валентной оболочке.

    Два примера:
    1. Из-за отсутствия d-орбиталей в его валентной оболочке фтор не может расширить свой октет, поэтому проявляет только степень окисления -1, в то время как все другие галогены из-за наличия d-орбиталей показывает положительные степени окисления +1, +3, +5 и +7, кроме степени окисления -1.

    2. Из-за небольшого размера три неподеленные пары электронов на каждом атоме F в молекуле F – F отталкивают пару связи. В результате энергия разрыва связи F-F ниже, чем у связи Cl-Cl.

    Вопрос 33.
    Почему реакционная способность азота отличается от фосфора?
    Ответ:
    Азот существует в виде двухатомной молекулы (N \(\equiv\) N). Из-за наличия тройной связи между двумя атомами азота энергия разрыва связи велика (941,4 кДж моль -1 ). В результате говорят, что азот химически инертен в своем элементарном состоянии. Напротив, одинарная связь P – P намного слабее (213 кДж моль -1 ), чем тройная связь N\(\эквив\)N. Поэтому фосфор гораздо более реакционноспособен, чем азот.

    Вопрос 34.
    Почему NH 3 образует водородную связь, а PH 3 нет?
    Ответ:
    Электроотрицательность N (3.0) значительно выше, чем у FI (2.1). В результате связь N–H достаточно полярна, и, следовательно, NH 3 образует межмолекулярную Н-связь. Напротив, и P, и H имеют электроотрицательность 2,1. Следовательно, связь P-H неполярна и, следовательно, PH 3 не образует H-связей.

    Вопрос 35.
    Кан PCl 5 действует как окислитель, а также как восстановитель? Оправдывать.
    Ответ:
    Степень окисления P в PCl 5 равна +5. Поскольку P имеет пять валентных электронов в своей валентной оболочке, поэтому он не может увеличить свою степень окисления выше +5, отдавая свои электроны, поэтому PCl 5 не может действовать как восстановитель. Однако он может понизить свою степень окисления с +5 до +3 или несколько ниже, поэтому PCl 5 действует как окислитель. Например, он окисляет Ag до AgCl.

    Samacheer Kalvi 12th Chemistry p-Block Elements – II 5 баллов Вопросы и ответы

    Вопрос 1.
    Как аммиак реагирует с

    1. Избыток Cl 2
    2. Нет
    3. CuSO 4
    4. О 2 / ∆

    Ответ:
    1. Реакция с избытком Cl 2

    2. Реакция с Na:

    3. Реакция с CuSO 4

    9.0002 Реакция с O 9.0008 4

    0008 2

    Вопрос 2.
    Объясните реакцию металлов с азотной кислотой.
    Ответ:
    Реакции металлов с азотной кислотой объясняются в 3 этапа следующим образом:
    Первичная реакция:
    Нитрат металла образуется с выделением выделяющегося водорода
    M + HNO 3 → MNO 3 + (H)

    Вторичная реакция:
    Возникающий водород образует продукты восстановления азотной кислоты.

    Третичная реакция:
    Вторичные продукты либо разлагаются, либо реагируют с образованием конечных продуктов

    Например:
    Медь реагирует с азотной кислотой следующим образом 3HNO 2 + 3H 2 O
    3HNO 2 → HNO 3 + 2NO + H 2 O

    Overall reaction
    3Cu + 8HNO 3 → 3CU(NO 3 ) 2 + 2NO + 4H 2 O

    Концентрированная кислота имеет тенденцию к образованию диоксида азота
    Cu + 4HNO 3 → 3CU(NO 3 ) 2 + 2NO 2 + 2H 2 O

    1 17 Каким методом вы будете готовить лабораторию? Объясните строение озона.
    Ответ:
    В лаборатории озон получают пропусканием электрического разряда через кислород. При потенциале 20 000 В около 10 % кислорода превращается в озон и дает смесь, известную как озонированный кислород. Чистый озон получают в виде бледно-голубого газа фракционной перегонкой сжиженного озонированного кислорода.


    Структура озона:
    Молекула озона имеет изогнутую форму и симметрична с делокализованными связями между атомами кислорода.

    Вопрос 4.
    А — царь кислоты. A реагирует с HBr с образованием B и брома. A реагирует с Na 2 CO 3 с образованием C и диоксида углерода. Определите A, B и C. Дайте реакцию.
    Ответ:
    1. Королем кислот является серная кислота (А).

    2. Серная кислота (A) реагирует с HBr с образованием SO 2 (B) и брома.

    3. Серная кислота (A) реагирует с Na2CO 3 с образованием сульфата натрия (C) и CO 2 .

    Вопрос 5.
    Как реагирует серная кислота следующим образом:
    (а) AL
    (B) KNO 3
    (C) NABR
    (D) C 6 H 6
    97 (D) C 6 H 6
    99997 (D) C 6 H 6
    99997 (D) C 6 H 6
    9997 (D) C 6 H 6
    999 (D) C 6 H 6

    (D) C 6
    (C)
    (D) C 3
    (C) Ответ:

    Вопрос 6.

    1. Объясните тест на сульфат (или) серную кислоту.
    2. Что происходит, когда серная кислота реагирует с щавелевой кислотой?

    Ответ:
    1. Разбавленный раствор серной кислоты или водный раствор сульфатов дает белый осадок с раствором хлорида бария. Его также можно обнаружить с помощью раствора ацетата свинца. При этом образуется белый осадок сернокислого свинца.

    2. Серная кислота реагирует с щавелевой кислотой с образованием CO и CO 2

    Вопрос 7.
    Обсудите производство хлора.
    Ответ:
    Электролитический процесс:
    При электролизе солевого раствора (NaCl) образуются ионы Na+ и CP. На 9Ион 0893 + реагирует с ионами OH воды с образованием гидроксида натрия. Водород и хлор выделяются в виде газов.

    Процесс Дикона:
    В этом процессе смесь воздуха и соляной кислоты пропускается через камеру, содержащую ряд полок, на которые помещаются пемзы, пропитанные хлоридом меди. Горячие газы при температуре около 723 К проходят через рубашку, окружающую камеру.

    Хлор, полученный этим методом, является разбавленным и используется для производства хлорной извести. Катализируемая реакция приведена ниже,

    Вопрос 8.

    1. Как готовят хлорную известь?
    2. Что происходит, когда бензол реагирует с хлором?

    Ответ:
    1. Хлорную известь получают пропусканием газообразного хлора через сухую гашеную известь (гидроксид кальция).

    2. Бензол реагирует с хлором в присутствии хлорида железа с образованием хлорбензола.

    Вопрос 9.
    Конус. H 2 SO 4 добавляют с последующим нагреванием в каждую из следующих пробирок, помеченных от (I) до (IV). Определите, в какой из приведенных выше пробирок будет наблюдаться следующее изменение. Подтвердите свой ответ химическим уравнением:
    (а) образование черного вещества
    (б) выделение коричневого газа
    (в) выделение бесцветного газа
    (г) образование коричневого вещества, которое при разбавлении становится синим.
    д) исчезновение желтого порошка с выделением бесцветного газа.
    Ответ:

    Вопрос 10.
    Укажите причины для каждого из следующих:

    1. Обесцвечивание цветов с помощью Cl 2 является постоянным, а с помощью SO 2 – временным.
    2. Расплавленный бромид алюминия является плохим проводником электричества.
    3. Оксид азота становится коричневым при попадании в воздух.
    4. PCl 5 имеет ионную природу в твердом состоянии.
    5. Аммиак является хорошим комплексообразователем.

    Ответ:
    1. Cl 2 отбеливает путем окисления, тогда как SO 2 делает это путем восстановления. Восстановленный продукт снова окисляется, и цвет восстанавливается.

    2. Бромид алюминия существует в виде димера Al 2 Br 6 . В этой структуре каждый атом алюминия образует одну координационную связь, принимая неподеленную пару электронов от атома брома другой молекулы бромида алюминия и, таким образом, завершая октет электронов. Из-за отсутствия свободных электронов расплавленный бромид алюминия является плохим проводником электричества.

    3. Оксид азота реагирует с воздухом и окисляется до NO 2 коричневого цвета.
    2no + o 2 → 2NO 2

    4. В твердотельном PCL 5 существует как [PCL 4 ] + [PCL 6 ] и там это природа.

  • Добавить комментарий

    Ваш адрес email не будет опубликован. Обязательные поля помечены *